ACCT 401 Test 2

¡Supera tus tareas y exámenes ahora con Quizwiz!

Which of the following are considered control environment factors? Detection Risk, Human Resources Policies and Practices A. Yes and Yes B. Yes and No C. No and Yes D. No and No

C. No and Yes

Early appointment of the auditor enables preliminary work to be performed by the auditor. This benefits the client because it permits the audit to be performed in A. A more efficient manner B. A more thorough manner C. Accordance with quality control standards D. Accordance with generally accepted auditing standards

A. A more efficient manner

Which of the following would not necessarily be a related party transaction? A. A sale to another corporation with a similar name. B. A purchase from another corporation that is controlled by the corporation's chief shareholder. C. Loan from the corporation to a major shareholder. D. Sale of land to the corporation by the spouse of a director.

A. A sales to another corporation with a similar name

A CPA is conducting the first audit of a nonissuer's financial statements. The CPA hopes to reduce the audit work by consulting with the predecessor auditor and reviewing the predecessor's audit documentation. This procedure is A. Acceptable if the client and the predecessor auditor agree to it. B. Acceptable if the CPA refers in the audit report to reliance upon the predecessor auditor's work as part of the basis for the CPA's own opinion. C. Required if the CPA is to express an unmodified opinion. D. Unacceptable because the CPA should bring an independent viewpoint to a new engagement.

A. Acceptable if the client and the predecessor auditor agree to it

The appropriate description that should be placed in symbol E is A. Accounts receivable master file. B. Bad debts master file. C. Remittance advice master file. D. Cash projection file.

A. Accounts receivable master file

Which of the following statements concerning noncompliance with laws and regulations by clients is correct? A. An auditor has responsibility to detect noncompliance with laws and regulations that has a direct effect on the financial statements. B. An audit in accordance with generally accepted auditing standards normally includes audit procedures specifically designed to detect noncompliance with laws and regulations having any effect on the financial statements. C. Whether an act constitutes noncompliance with laws and regulations is a matter of auditor judgment. D. An auditor has no responsibility to detect noncompliance with laws and regulations that has an effect on the financial statements.

A. An auditor has responsibility to detect noncompliance with laws and regulations that has a direct effect on the financial statements

What is the definition of fraud in an audit of financial statements? A. An intentional act that results in a material misstatement in financial statements that are the subject of an audit. B. The unintentional misapplication of accounting principles relating to amounts, classification, manner of presentation, or disclosure. C. An intentional act that results in a material weakness in financial statements that are the subject of an audit. D. Management's inability to design and implement programs and controls to prevent, deter, and detect material misstatements.

A. An intentional act that results in a material misstatement in financial statements that are the subject of an audit

The acceptable level of detection risk is inversely related to the A. Assurance provided by substantive procedures. B. Risk of misapplying auditing procedures. C. Preliminary judgment about materiality levels. D. Risk of failing to discover material misstatements.

A. Assurance provided by substantive procedures

Analytical procedures are required for which of the following? A. Audit planning. B. Tests of balances. C. Client retention decision. D. Internal control evaluation.

A. Audit planning

Which of the following statements describes why a properly planned and performed audit may not detect a material misstatement due to fraud? A. Audit procedures that are effective for detecting an error may be ineffective for detecting fraud that is concealed through collusion. B. An audit is designed to provide reasonable assurance of detecting material errors, but there is no similar responsibility concerning material fraud. C. The factors considered in assessing the risk of material misstatement indicated an increased risk of intentional misstatements, but only a low risk of errors in the financial statements. D. The auditor did not consider factors influencing audit risk for account balances that have effects pervasive to the financial statements as a whole.

A. Audit procedures that are effective for detecting an error may be ineffective for detecting fraud that is concealed through collusion

Which of the following elements ultimately determines the specific auditing procedures that are necessary in the circumstances to provide a reasonable basis for an opinion? A. Auditor judgment. B. Materiality. C. Audit risk. D. Reasonable assurance.

A. Auditor judgment

Audit risk at the assertion level consists of inherent risk, control risk, and detection risk. Which of the following statements is true? A. Cash has a greater inherent risk than an inventory of coal because it is more susceptible to theft. B. The risk that material misstatement will not be timely prevented or detected by internal control can be reduced to zero by effective controls. C. Detection risk is a function of the efficiency of an auditing procedure. D. The existing levels of inherent risk, control risk, and detection risk can be changed at the discretion of the auditor.

A. Cash has a greater inherent risk than an inventory of coal because it is more susceptible to theft.

Which of the following statements indicates the wrong way to use an internal control questionnaire? A. Clarifying all answers with written remarks and explanations. B. Filling out the questionnaire during an interview with the person who has responsibility for the area that is being audited. C. Constructing the questionnaire so that a no response requires attention. D. Supplementing the completed questionnaire with a narrative description or flowchart.

A. Clarifying all answers with written remarks and explanations

An auditor's audit documentation will least likely show how the A. Client's schedules were prepared. B. Engagement was planned. C. Understanding of the client's internal control was obtained and the risks of material misstatement were assessed. D. Significant issues were resolved.

A. Client's schedules were prepared

If the auditors plan to use the work of the internal auditors to obtain audit evidence or to provide direct assistance, they should assess the internal auditors' A. Competence and objectivity. B. Efficiency and experience. C. Independence and review skills. D. Training and supervisory skills.

A. Competence and objectivity

Analytical procedures used to form an overall audit conclusion generally include A. Considering unusual or unexpected account balances that were not previously identified. B. Performing tests of transactions to corroborate management's financial statement assertions. C. Gathering evidence concerning account balances that have not changed from the prior year. D. Retesting controls that appeared to be ineffective during the assessment of control risk.

A. Considering unusual or unexpected account balances that were not previously identified

Certain individuals may have an attitude, character, or set of values that permit them to rationalize fraud. Moreover, individuals may have an incentive or be under pressure to commit fraud, or circumstances may provide an opportunity. The auditor's concern about the risk of material misstatements due to fraud is least likely to be increased if management A. Consists of many individuals that make operating and financing decisions. B. Commits to unduly aggressive forecasts. C. Has an excessive interest in increasing the entity's stock price through use of unduly aggressive accounting practices. D. Is interested in inappropriate means of minimizing reported earnings for tax-motivated reasons.

A. Consists of many individuals that make operating and financing decisions

Which result of an analytical procedure suggests the existence of obsolete merchandise? A. Decrease in the inventory turnover rate. B. Decrease in the ratio of gross profit to sales. C. Decrease in the ratio of inventory to accounts payable. D. Decrease in the ratio of inventory to accounts receivable.

A. Decrease in the inventory turnover rate

Internal auditing can affect the scope of the external auditor's audit of financial statements by A. Decreasing the external auditor's need to perform detailed tests. B. Allowing the external auditor to limit his or her audit to substantive testing. C. Limiting direct testing by the external auditor to assertions not directly tested by internal auditing. D. Eliminating the need to be on hand during the physical count of inventory.

A. Decreasing the external auditor's need to perform detailed tests

Each of the following might, by itself, form a valid basis for an auditor to decide to omit a procedure except for the A. Difficulty and cost involved in testing a particular item. B. Assessment of the risks of material misstatement at a low level. C. Inherent risk involved. D. Relationship between the cost of obtaining evidence and its usefulness.

A. Difficulty and cost involved in testing a particular item

Which action regarding fraud is an activity related to performance of risk assessment procedures? A. Discussions among the engagement team members regarding the risks of material misstatement due to fraud. B. Document the results of procedures used to address the risk of fraud. C. Consider the characteristics of journal entries, particularly those made near year end. D. Consider whether estimates prepared and recorded by management could indicate a bias in reporting.

A. Discussions among the engagement team members regarding the risks of material misstatement due to fraud

Which of the following would be least likely to be comparable between similar corporations in the same industry line of business? A. Earnings per share. B. Return on total assets before interest and taxes. C. Accounts receivable turnover. D. Operating cycle.

A. Earnings per share

Standardized working papers are often used, chiefly because they allow audit documentation to be prepared more A. Efficiently. B. Professionally. C. Neatly. D. Accurately.

A. Efficiently

Directors, management, external auditors, and internal auditors all play important roles in creating proper control processes. Senior management is primarily responsible for A. Establishing risk management and control processes. B. Reviewing the reliability and integrity of financial and operational information. C. Ensuring that external and internal auditors oversee the administration of risk management and control processes. D. Implementing and monitoring controls designed by the board of directors.

A. Establishing risk management and control processes

An auditor discovers that a client's accounts receivable turnover is substantially lower for the current year than for the prior year. This trend may indicate that A. Fictitious credit sales have been recorded during the year. B. Employees have stolen inventory just before year end. C. The client recently tightened its credit-granting policies. D. An employee has been lapping receivables in both years.

A. Fictitious credit sales have been recorded during the year

Which of the following is an aspect of scheduling and controlling the audit engagement? A. Include in the audit plan a column for estimated and actual time. B. Perform audit work only after the client's books of account have been closed for the period under examination. C. Write a conclusion in the audit documentation indicating how the results of the audit will affect the auditor's report. D. Include in the engagement letter an estimate of the minimum and maximum audit fee.

A. Include in the audit plan a column for estimated and actual time

Which of the following most likely would not be considered an inherent limitation of the potential effectiveness of an entity's internal control? A. Incompatible duties. B. Management override. C. Faulty judgment. D. Collusion among employees.

A. Incompatible duties

In assessing the objectivity of internal auditors, the independent CPA who is auditing the entity's financial statements most likely considers the A. Internal auditing standards developed by The Institute of Internal Auditors. B. Tests of internal control activities that could detect errors and fraud. C. Materiality of the accounts recently inspected by the internal auditors. D. Results of the tests of transactions recently performed by the internal auditors.

A. Internal auditing standards developed by The Institutes of Internal Auditors

A CPA's understanding of internal control in a financial statement audit of a nonissuer A. Is usually more limited than that made in an audit of internal control integrated with an audit of financial statements. B. Is usually more extensive than that made in an audit of internal control integrated with an audit of financial statements. C. Will usually be identical to that made in an audit of internal control integrated with an audit of financial statements. D. Will usually result in a report on the effectiveness of internal control.

A. Is usually more limited than that made in an audit of internal control integrated with an audit of financial statements

The appropriate description that should be placed in symbol B is A. Keying and verifying. B. Error correction. C. Collation of remittance advices. D. Batch processing.

A. Keying and verifying

The negative request form of accounts receivable confirmation may be used when the Risk of Material Misstatement is, Number of Small Balances is, Consideration by the Recipient is A. Low, Many, Likely B. Low, Few, Unlikely C. High, Few, Likely D. High, Many, Likely

A. Low, Many, Likely

Before accepting an engagement to audit a new client, an auditor is required to A. Making inquiries of the predecessor auditor after obtaining the consent of the prospective client B. Obtain the prospective client's signature to the engagement letter C. Prepare a memorandum setting forth the staffing requirements and documenting the preliminary audit plan D. Discuss the management representation letter with the prospective client's audit committee.

A. Making inquiries of the predecessor auditor after obtaining the consent of the prospective client

A small private entity may use less formal means to ensure that internal control objectives are achieved. For example, extensive accounting procedures, sophisticated accounting records, or formal controls are least likely to be needed if A. Management is closely involved in operations. B. The entity is involved in complex transactions. C. The entity is subject to legal or regulatory requirements also found in larger entities. D. Financial reporting objectives have been established.

A. Management is closely involved in operations

An auditor's engagement letter most likely will include A. Management's acknowledgment of its responsibility for maintaining effective internal control. B. The auditor's preliminary assessment of the risk factors relating to misstatements arising from fraudulent financial reporting. C. A reminder that management is responsible for illegal acts committed by employees. D. A request for permission to contact the client's lawyer for assistance in identifying litigation, claims, and assessments.

A. Management's acknowledgment of its responsibility for maintaining effective internal control.

Internal controls are designed to provide reasonable assurance that A. Material errors or fraud will be prevented, or detected and corrected, within a timely period by employees in the course of performing their assigned duties. B. Management's plans have not been circumvented by worker collusion. C. The internal auditing department's guidance and oversight of management's performance is accomplished economically and efficiently. D. Management's planning, organizing, and directing processes are properly evaluated.

A. Material errors or fraud will be prevented, or detected and corrected, within a timely period by employees in the course of performing their assigned duties

Madison Corporation has a few large accounts receivable that total $1,000,000. Nassau Corporation has a great number of small accounts receivable that also total $1,000,000. The importance of a misstatement in any one account is therefore greater for Madison than for Nassau. This is an example of the auditor's concept of A. Materiality. B. Comparative analysis. C. Reasonable assurance. D. Audit risk.

A. Materiality

If an auditor conducts an audit of financial statements in accordance with applicable auditing standards, which of the following will the auditor most likely detect? A. Misposting of recorded transactions. B. Unrecorded transactions. C. Forgery. D. Collusive fraud.

A. Misposting of recorded transactions

You have been assigned to audit the maintenance department of an organization. Which of the following is likely to produce the least reliable audit evidence? A. Notes on discussions with mechanics in the maintenance operation. B. A schedule comparing actual maintenance expenses with budgeted expenses and those of the prior period and disclosing important differences. C. A narrative covering review of user reports on maintenance service. D. An analysis of changes in certain maintenance department ratios.

A. Notes on discussions with mechanics in the maintenance operation

Which of the following procedures concerning accounts receivable is an auditor most likely to perform to obtain evidence in support of the effectiveness of controls? A. Observing an entity's employee prepare the schedule of past due accounts receivable. B. Sending confirmation requests to an entity's principal customers to verify the existence of accounts receivable. C. Inspecting an entity's analysis of accounts receivable for unusual balances. D. Comparing an entity's uncollectible accounts expense with actual uncollectible accounts receivable.

A. Observing an entity's employee prepare the schedule of past due accounts receivable

An auditor confirms a representative number of open accounts receivable as of December 31 and investigates respondents' exceptions and comments. By this procedure, the auditor would be most likely to learn of which of the following? A. One of the cashiers has been covering a personal embezzlement by lapping. B. One of the sales clerks has not been preparing charge slips for credit sales to family and friends. C. One of the computer control clerks has been removing all sales invoices applicable to his account from the data file. D. The credit manager has misappropriated remittances from customers whose accounts have been written off.

A. One of the cashiers have been covering a personal embezzlement by lapping

Audit working papers are indexed by means of reference numbers. The primary purpose of indexing is to A. Permit cross-referencing and simplify supervisory review. B. Support the audit report. C. Eliminate the need for follow-up reviews. D. Determine that working papers adequately support findings, conclusions, and reports.

A. Permit cross-referencing and simplify supervisory review

An internal auditor is reviewing the following computer logic diagram: Basic Flowchart This diagram represents which of the following? A. Program loop step. B. Data validity check. C. Balance test. D. Limit test.

A. Program loop step

In assessing the competence of an internal auditor, an independent CPA most likely would obtain information about the A. Quality of the internal auditor's documentation. B. Organization's commitment to integrity and ethical values. C. Influence of management on the scope of the internal auditor's duties. D. Organizational levels to which the internal auditor reports.

A. Quality of the internal auditor's documentation

Effective internal control A. Reduces the need for management to review exception reports on a day-to-day basis. B. Eliminates risk and potential loss to the organization. C. Cannot be circumvented by management. D. Is unaffected by changing circumstances and conditions encountered by the organization.

A. Reduces the need for management to review exception reports on a day-to-day basis

An auditor would be most likely to consider modifying an otherwise unmodified opinion if the client's financial statements include a note on related party transactions A. Representing without substantiation that certain related party transactions were consummated on terms equivalent to those obtainable in transactions with unrelated parties. B. Presenting the dollar volume of related party transactions and the effects of any change in the method of establishing terms from that used in the prior period. C. Explaining the business purpose of the sale of real property to a related party. D. Disclosing compensating balance arrangements maintained for the benefit of related parties.

A. Representing without substantiation that certain related party transactions were consummated on terms equivalent to those obtainable in transactions with unrelated parties

In the confirmation of accounts receivable, the auditor would most likely A. Request confirmation of a sample of the inactive accounts. B. Seek to obtain positive confirmations for at least 50% of the total dollar amount of the receivables. C. Require confirmation of all receivables from agencies of the federal government. D. Require that confirmation requests be sent within 1 month of the fiscal year end.

A. Request confirmation of a sample of the inactive accounts

The PCAOB's AS 2201 states that internal controls may be preventive or detective. Which of the following controls is preventive? A. Requiring two persons to open mail. B. Reconciling the accounts receivable subsidiary file with the control account. C. Using batch totals. D. Preparing bank reconciliations.

A. Requiring two persons to open mail

Which of the following statements about the auditor's response to assessed risks of material misstatement in a financial statement audit is true? A. Risk assessment procedures performed to obtain an understanding of an entity's internal control also may serve as tests of controls. B. When the risks of material misstatement are high, an auditor should reduce the amount of substantive testing. C. Reliance on internal control may be sufficient to allow the auditor to eliminate substantive testing for significant transaction classes. D. When assessing the risks of material misstatement, an auditor should not consider evidence obtained in prior audits about the operation of controls.

A. Risk assessment procedures performed to obtain an understanding of an entity's internal control also may serve as tests of controls

Basic to a proper control environment are the quality and integrity of personnel who must perform the prescribed procedures. Which is not a factor in providing for competent personnel? A. Segregation of duties. B. Hiring practices. C. Training programs. D. Performance evaluations.

A. Segregation of duties

One of the primary roles of an engagement work program is to A. Serve as a tool for planning and conducting engagement work. B. Document an internal auditor's evaluations of controls. C. Provide for a standardized approach to the engagement. D. Assess the risks associated with the activity under review.

A. Serve as a tool for planning and conducting engagement work

Analytical procedures can best be categorized as A. Substantive procedures. B. Tests of controls. C. Qualitative tests. D. Budget comparisons.

A. Substantive procedures

The auditor should perform tests of controls when the auditor's assessment of the risks of material misstatement includes an expectation of the operating effectiveness of internal control or when A. Substantive procedures alone cannot provide sufficient appropriate audit evidence at the relevant assertion level. B. Tests of details and substantive analytical procedures provide sufficient appropriate audit evidence to support the assertion being evaluated. C. The auditor is not able to obtain an understanding of internal controls. D. The owner-manager performs virtually all the functions of internal control.

A. Substantive procedures alone cannot provide sufficient appropriate audit evidence at the relevant assertion level

A client uses a suspense account for unresolved questions whose final accounting has not been determined. If a balance remains in the suspense account at year-end, the auditor would be most concerned about A. Suspense debits that management believes will benefit future operations. B. Suspense debits that the auditor verifies will have realizable value to the client. C. Suspense credits that management believes should be classified as "current liability." D. Suspense credits that the auditor determines to be customer deposits.

A. Suspense debits that management believes will benefit future operations

When an auditor plans to rely on controls that have changed since they were last tested, which of the following courses of action would be most appropriate? A. Test the operating effectiveness of such controls in the current audit. B. Document that reliance and proceed with the original audit strategy. C. Inquire of management as to the effectiveness of the controls. D. Report the reliance in the report on internal controls.

A. Test the operating effectiveness of such controls in the current audit

The client has equity securities classified as available for sale. The auditor is most concerned about controls related to A. The determination of the fair value measurements of the securities. B. The accrued interest receivable for the securities. C. Why specific securities were purchased. D. When the securities will be sold.

A. The determination of the fair value measurements of the securities

Which of the following characteristics most likely would heighten an auditor's concern about the risk of material misstatements arising from fraudulent financial reporting? A. The entity's industry is experiencing declining customer demand. B. Employees who handle cash receipts are not bonded. C. Bank reconciliations usually include in-transit deposits. D. Equipment is often sold at a loss before being fully depreciated.

A. The entity's industry is experiencing declining customer demand

A client decides not to correct misstatements communicated by the auditor that collectively are not material and wants the auditor to issue the report based on the uncorrected numbers. Which of the following statements is correct regarding the financial statement presentation? A. The financial statements are free from material misstatement, and no disclosure is required in the notes to the financial statements. B. The financial statements are not in accordance with the applicable financial reporting framework. C. The financial statements contain uncorrected misstatements that should result in a qualified opinion. D. The financial statements are free from material misstatement, but disclosure of the proposed adjustments is required in the notes to the financial statements.

A. The financial statements are free from material misstatement, and no disclosure is required in the notes to the financial statements.

Which of the following is a false statement about the relationship of financial statement assertions and audit procedures? A. The relationship between financial statement assertions and audit procedures should be one-to-one. B. Audit procedures should be developed in light of financial statement assertions about the financial statement components. C. Selection of tests of financial statement assertions should depend upon the understanding of internal control. D. The auditor should resolve any substantial doubt about any of management's relevant financial statement assertions.

A. The relationship between financial statement assertions and audit procedures should be one-to-one

The normal sequence of documents and operations on a well-prepared systems flowchart is A. Top to bottom and left to right. B. Bottom to top and left to right. C. Top to bottom and right to left. D. Bottom to top and right to left.

A. Top to bottom and left to right

Analytical procedures enable the auditor to predict the balance or quantity of an item under audit. Information to develop this estimate can be obtained from all of the following except A. Tracing transactions through the system to determine whether procedures are being applied as prescribed. B. Comparison of financial data with data for comparable prior periods, anticipated results (e.g., budgets and forecasts), and similar data for the industry in which the entity operates. C. Study of the relationships of elements of financial data that would be expected to conform to a predictable pattern based upon the entity's experience. D. Study of the relationships of financial data with relevant nonfinancial data.

A. Tracing transactions through the system to determine whether procedures are being applied as prescribed

The primary difference between an audit of the balance sheet and an audit of the income statement is that the audit of the income statement deals with the verification of A. Transactions. B. Authorizations. C. Costs. D. Cutoffs.

A. Transactions

The objective of performing analytical procedures in planning an audit is to identify the existence of A. Unusual transactions and events. B. Noncompliance with laws and regulations that went undetected because of internal control deficiency. C. Related party transactions. D. Recorded transactions that were not properly authorized.

A. Unusual transactions and events

The symbol used to represent the physical act of collecting employees' time cards for processing is A. Upside Down Trapezoid B. Rectangle with a Slanted Top C. Rectangle D. Slanted Rectangle

A. Upside Down Trapezoid

Which of the following factors has the least influence on an auditor's consideration of the reliability of data for purposes of analytical procedures? A. Whether the data were processed in a computer system or in a manual accounting system. B. Whether sources within the entity were independent of those who are responsible for the amount being audited. C. Whether the data were subjected to audit testing in the current or prior year. D. Whether the data were obtained from independent sources outside the entity or from sources within the entity.

A. Whether the data were processed in a computer system or in a manual accounting system

After audit procedures are completed, a partner of the CPA firm who has not been involved in the audit performs a second or wrap-up audit documentation review. This second review usually focuses on A. Whether the financial statements are consistent with the auditor's understanding of the entity. B. Fraud involving the client's management and its employees. C. The materiality of the adjusting entries proposed by the audit staff. D. The communication of internal control weaknesses to those charged with governance.

A. Whether the financial statements are consistent with the auditor's understanding of the entity

In assessing whether to accept a client for an audit engagement, a CPA should consider the Client's Business Risk, CPA's Business Risk A. Yes and Yes B. Yes and No C. No and Yes D. No and No

A. Yes and Yes

Ordinarily, the predecessor auditor permits the auditor to review the predecessor's audit documentation relating to Contingencies, Balance Sheet Accounts A. Yes and Yes B. Yes and No C. No and Yes D. No and No

A. Yes and Yes

For all audits of financial statements made in accordance with generally accepted auditing standards, the auditor should apply analytical procedures to some extent as Risk Assessment Procedures, Substantive Procedures, In the Review Stage A. Yes, No, and Yes B. No, Yes, and No C. No, No, and Yes D. Yes, Yes, and No

A. Yes, No, and Yes

Which of the following audit risk components may be assessed in nonquantitative terms? Control Risk, Detection Risk, Inherent Risk A. Yes, Yes, and Yes B. No, Yes, and Yes C. Yes, Yes, and No D. Yes, No, and Yes

A. Yes, Yes, and Yes

The primary purpose of obtaining an understanding of the entity and its environment, including its internal control, is to provide an auditor with A. Evidence to use in reducing detection risk. B. A frame of reference within which to plan the audit. C. A basis for modifying tests of controls. D. Information necessary to prepare flowcharts.

B. A frame of reference within which to plan the audit

Which of the following situations represents a risk factor that relates to misstatements arising from misappropriation of assets? A. A high turnover of senior management. B. A lack of independent checks. C. A strained relationship between management and the predecessor auditor. D. An inability to generate cash flow from operations.

B. A lack of independent checks

The components of internal control include A. Monitoring of controls that sets the tone of the organization. B. A process of managing risks relevant to preparing financial statements. C. A control environment consisting of policies and procedures to help ensure that management directives are carried out. D. Control activities that identify, capture, and exchange information.

B. A process of managing risks relevant to preparing financial statements

A CPA wishes to determine how various issuers have complied with the disclosure requirements of a new financial accounting standard. Which of the following information sources would the CPA most likely consult for this information? A. AICPA Codification of Statements on Auditing Standards. B. AICPA Accounting Trends and Techniques. C. PCAOB Inspection Reports. D. SEC Statement 10-K Guide.

B. AICPA Accounting Trends and Techniques

For the fiscal year ending December 31 of the previous year and for the current year, Justin Co. has net sales of $1,000,000 and $2,000,000; average gross receivables of $100,000 and $300,000; and an allowance for uncollectible accounts receivable of $30,000 and $50,000, respectively. If the accounts receivable turnover and the ratio of allowance for uncollectible accounts receivable to gross accounts receivable are calculated, which of the following best represents the conclusions to be drawn? A. Accounts receivable turnovers are 10.0 and 6.6, and the ratios of uncollectible accounts receivable to gross accounts receivable are 0.30 and 0.16, respectively. Examine allowance for possible overstatement of the allowance. B. Accounts receivable turnovers are 10.0 and 6.7, and the ratios of uncollectible accounts receivable to gross accounts receivable are 0.30 and 0.17, respectively. Examine allowance for possible understatement of the allowance. C. Accounts receivable turnovers are 14.3 and 8.0, and the ratios of uncollectible accounts receivable to gross accounts receivable are 0.42 and 0.20, respectively. Examine allowance for possible overstatement of the allowance. D. Accounts receivable turnovers are 14.3 and 8.0 and the ratios of uncollectible accounts receivable to gross accounts receivable are 0.42 and 0.20, respectively. Examine allowance for possible understatement of the allowance.

B. Accounts receivable turnovers are 10.0 and 6.7, and the ratios of uncollectible accounts receivable to gross accounts receivable are 0.30 and 0.17, respectively. Examine allowance for possible understatement of the allowance.

It is important for the auditor to consider the competence of the audit client's employees, because their competence bears directly and importantly upon the A. Relationship of the costs of internal control and its benefits. B. Achievement of the objectives of internal control. C. Comparison of recorded accountability with assets. D. Timing of the tests to be performed.

B. Achievement of the objectives of internal control

In an audit of financial statements, an auditor's primary consideration regarding an internal control is whether the control A. Reflects management's philosophy and operating style. B. Affects management's financial statement assertions. C. Provides adequate safeguards over access to assets. D. Relates to operational objectives.

B. Affects management's financial statement assertions

Financial statement audit plans usually should be developed A. Prior to performing risk assessment procedures. B. After the auditor has established the overall audit strategy. C. After obtaining an understanding of the information and communication and control activities components of internal control. D. When the engagement letter is prepared.

B. After the auditor has established the overall audit strategy

Which of the following is required documentation in an audit in accordance with auditing standards? A. A flowchart or narrative of information system describing the recording and classification of transactions for financial reporting B. An audit plan documenting the procedures to be used to reduce audit risk C. A planning memorandum establishing the timing of the audit procedures and coordinating the assistance of entity personnel D. An internal control questionnaire identifying policies and procedures that assure specific objectives will be achieved

B. An audit plan documenting the procedures to be used to reduce audit risk

Which of the following presumptions is least likely to relate to the reliability of audit evidence? A. The more effective internal control is, the more assurance it provides about the accounting data and financial statements. B. An auditor's opinion is formed within a reasonable time to achieve a balance between benefit and cost. C. Evidence obtained from independent sources outside the entity is more reliable than evidence secured solely within the entity. D. The auditor's direct personal knowledge obtained through observation and inspection is more persuasive than information obtained indirectly.

B. An auditor's opinion is formed within a reasonable time to achieve a balance between benefit and cost

The following is a section of a system flowchart for a payroll application: Flowchart with X in the Rectangle with Squiggly Line at the Bottom Symbol X could represent A. Erroneous time cards. B. An error report. C. Batched time cards. D. Unclaimed payroll checks.

B. An error report

Which of the following statements is true concerning analytical procedures used as risk assessment procedures? A. Analytical procedures usually involve comparisons of ratios developed from recorded amounts with assertions developed by management. B. Analytical procedures used as risk assessment procedures ordinarily use data aggregated at a high level. C. Analytical procedures can replace tests of controls in gathering evidence to support the assessed level of control risk. D. Analytical procedures are more efficient, but not more effective, than tests of details and transactions.

B. Analytical procedures used as risk assessment procedures ordinarily use data aggregated at a high level

Transaction authorization within an organization may be either specific or general. An example of specific transaction authorization is the A. Setting of automatic reorder points for material or merchandise. B. Approval of a detailed construction budget for a warehouse. C. Establishment of requirements to be met in determining a customer's credit limits. D. Establishment of sales prices for products to be sold to any customer.

B. Approval of a detailed construction budget for a warehouse

Using personal computers in auditing may affect the methods used to review the work of staff assistants because A. Supervisory personnel may not have an understanding of the capabilities and limitations of personal computers. B. Audit documentation may not contain readily observable details of calculations. C. The audit standards for supervision may differ. D. Documenting the supervisory review may require assistance of consulting services personnel.

B. Audit documentation may not contain readily observable details of calculations

An auditor's decision either to apply analytical procedures as substantive procedures or to perform tests of transactions and account balances usually is determined by the A. Availability of data aggregated at a high level. B. Auditor's determination about whether audit risk can be sufficiently reduced. C. Timing of tests performed after the balance sheet date. D. Auditor's familiarity with industry trends.

B. Auditor's determination about whether audit risk can be sufficiently reduced

The PCAOB's AS 1215, Audit Documentation, requires that a complete and final set of audit documentation be assembled for retention as of a date not more than 45 days after the audit report release date. After that date, audit documentation may A. Be deleted or discarded, but no information may be added. B. Be added, but no audit documentation may be deleted or discarded. C. Not be deleted, discarded, or added. D. Be deleted, discarded, or added as necessary.

B. Be added, but no audit documentation may be deleted or discarded

The auditor's understanding of internal control is documented to substantiate A. Conformity of the accounting records with generally accepted accounting principles. B. Compliance with generally accepted auditing standards. C. Adherence to procedures for effective and efficient management decision making. D. The fairness of the financial statement presentation.

B. Compliance with generally accepted auditing standards

Which of the following is an unusual procedure that may be deemed necessary to discover the effect of a related party transaction? A. Examine invoices and other pertinent documents such as receiving or shipping reports. B. Confirm significant information with third parties other than banks or attorneys. C. Determine whether the transaction has been approved by management, those charged with governance, or shareholders. D. Inspect or confirm the transferability and value of collateral.

B. Confirm significant information with third parties other than banks or attorneys

An auditor's consideration of the risk of material misstatement due to fraud and the results of audit tests indicate a significant risk of fraud. The auditor should A. Express either a qualified or an adverse opinion. B. Consider withdrawing from the engagement and communicating the reasons for withdrawal to those charged with governance. C. Express only an adverse opinion because of the strong possibility of fraud. D. Inform proper authorities outside the entity.

B. Consider withdrawing from the engagement and communicating the reasons for withdrawal to those charged with governance

An auditor is assessing the appropriateness of management's rationale for selecting a model to measure the fair value of debt securities. If, during the current year, an active trading market for the debt security was introduced, the auditor should validate each of the following criteria, except whether the valuation model is A. Appropriate for the environment in which the entity operates. B. Consistently applied from prior periods. C. Evaluated and appropriately applied based on generally accepted accounting principles. D. Appropriate for the debt security being valued.

B. Consistently applied from prior periods

An auditor's preliminary analysis of accounts receivable turnover revealed the following rates over these accounting periods: Year 3 Year 2 Year 1 4.3 6.2 7.3 Which of the following is the most likely cause of the decrease in accounts receivable turnover? A. Increase in the cash discount offered. B. Liberalization of credit policy. C. Shortening of due date terms. D. Increased cash sales.

B. Liberalization of credit policy

If the auditor considers an act of noncompliance with laws and regulations to be sufficiently serious to warrant withdrawing from the engagement, the auditor would likely A. Notify all parties who may rely upon the company's financial statements of the company's illegal act. B. Consult with legal counsel as to what other action, if any, should be taken. C. Return all incriminating evidence and working papers to the client's audit committee for follow-up. D. Contact the successor auditor to make the successor aware of the possible consequences of relying on management's representations.

B. Consult with legal counsel as to what other action, if any, should be taken

Which of the following best describe the interrelated components of internal control? A. Organizational structure, management philosophy, and planning. B. Control environment; risk assessment process; control activities; the information system, including related business processes; and monitoring of controls. C. Risk assessment process, backup facilities, responsibility accounting, and natural laws. D. Assignment of authority and responsibility, management philosophy, and organizational structure

B. Control environment; risk assessment process; control activities; the information system, including related business processes; and monitoring of controls

Which one of the following best reflects the basic elements of a data flow diagram? A. Data sources, data flows, computer configurations, flowchart, and data storage. B. Data source, data destination, data flows, transformation processes, and data storage. C. Data flows, data storage, and program flowchart. D. Data flows, program flowchart, and data destination.

B. Data source, data destination, data flows, transformation processes, and data storage

Under the Private Securities Litigation Reform Act of 1995, Baker, CPA, reported certain noncompliance with laws and regulations to Supermart's board of directors. Baker believed that failure to take remedial action would warrant a qualified audit opinion because the noncompliance had a material effect on Supermart's financial statements. Supermart failed to take appropriate remedial action, and the board of directors refused to inform the SEC that it had received such notification from Baker. Under these circumstances, Baker is required to A. Resign from the audit engagement within 10 business days. B. Deliver a report concerning the noncompliance to the SEC within 1 business day. C. Notify the shareholders that the financial statements are materially misstated. D. Withhold an audit opinion until Supermart takes appropriate remedial action.

B. Deliver report concerning the noncompliance to the SEC within 1 business day

When planning an audit, an auditor should A. Consider whether substantive procedures may be reduced based on the results of the internal control questionnaire. B. Determine materiality for the financial statements as a whole. C. Conclude whether changes in compliance with prescribed controls require a change in the reliance on controls. D. Prepare a preliminary draft of the management representation letter.

B. Determine materiality for the financial statements as a whole

In obtaining an understanding of internal control, the auditor may trace several transactions through the control process, including how the transactions interface with any service organizations whose services are part of the information system. The primary purpose of this task is to A. Replace substantive procedures. B. Determine whether the controls have been implemented. C. Determine the effectiveness of the control procedures. D. Detect fraud.

B. Determine whether the controls have been implemented

The symbol employed to determine if an employee's wages are above or below the maximum limit for FICA taxes is A. Rectangle B. Diamond C. Upside Down Trapezoid D. Square Open on the Right Side

B. Diamond

Which of the following circumstances most likely will cause an auditor to suspect that material misstatements exist in a client's financial statements? A. The assumptions used in developing the prior year's accounting estimates have changed. B. Differences between reconciliations of control accounts and subsidiary records are not investigated. C. Negative confirmation requests yield fewer responses than in the prior year's audit. D. Management consults with another CPA firm about complex accounting matters.

B. Differences between reconciliations of control accounts and subsidiary records are not investigated

The auditor with final responsibility for an engagement and one of the assistants have a difference of opinion about the results of an auditing procedure. If the assistant believes it is necessary to be disassociated from the matter's resolution, the CPA firm's procedures should enable the assistant to A. Refer the disagreement to the AICPA's Peer Review Board B. Document the details of the disagreement with the conclusion reached C. Discuss the disagreement with the entity's management or its audit committee D. Report the disagreement to an impartial peer review monitoring team

B. Document the details of the disagreement with the conclusion reached

Internal control cannot be designed to provide reasonable assurance regarding the achievement of objectives concerning A. Reducing the cost of an external audit. B. Elimination of all fraud. C. Availability of reliable data for decision-making purposes and protection of important documents and records. D. Compliance with the Foreign Corrupt Practices Act of 1977.

B. Elimination of all fraud

The next action to take with the customer remittance advices (refer to Symbol C) would be to A. Discard them immediately. B. File them daily by batch number. C. Forward them to the internal audit department for internal review. D. Forward them to the treasurer to compare with the monthly bank statement.

B. File them daily by batch number

The in-charge auditor for an audit of an issuer most likely has a supervisory responsibility to explain to the staff assistants A. That immaterial fraud is not to be reported to the client's audit committee. B. How the results of various auditing procedures performed by the assistants should be evaluated. C. What benefits may be attained by the assistants' adherence to established time budgets. D. Why certain documents are being transferred from the current file to the permanent file.

B. How the results of various auditing procedures performed by the assistants should be evaluated

Which of the following steps should an auditor perform first to determine the existence of related parties? A. Examine invoices, contracts, and purchasing orders. B. Inquire about the existence of related parties from management. C. Review the company's business structure. D. Review proxy and other materials filed with the SEC.

B. Inquire about the existence of related parties from management

Decision tables differ from program flowcharts in that decision tables emphasize A. Ease of manageability for complex programs. B. Logical relationships among conditions and actions. C. Cost-benefit factors justifying the program. D. The sequence in which operations are performed.

B. Logical relationships among conditions and actions

According to AU-C 315, Understanding the Entity and its Environment and Assessing the Risks of Material Misstatement, not all controls are relevant to a financial statement audit. Which one of the following would most likely be considered in an audit? A. Timely reporting and review of quality control results. B. Maintenance of control over unused checks. C. Marketing analysis of sales generated by advertising projects. D. Maintenance of statistical production analyses.

B. Maintenance of control over unused checks

Management's attitude toward aggressive financial reporting and its emphasis on meeting projected profit goals most likely will significantly increase opportunities for fraudulent financial reporting when A. External policies established by parties outside the entity affect its accounting practices. B. Management is dominated by one individual who is also a shareholder. C. Internal auditors have direct access to the board of directors and the entity's management. D. The audit committee is active in overseeing the entity's financial reporting policies.

B. Management is dominated by one individual who is also a shareholder

When obtaining an understanding of an entity's internal control, an auditor should concentrate on their substance rather than their form because A. The controls may be operating effectively but may not be documented. B. Management may establish appropriate controls but not enforce compliance with them. C. The controls may be so inappropriate that no reliance is expected by the auditor. D. Management may implement controls whose costs exceed their benefits.

B. Management may establish appropriate controls by not enforce compliance with them

The ultimate purpose of understanding internal control is to contribute to the auditor's evaluation of the risk that A. Tests of controls may fail to identify controls relevant to assertions. B. Material misstatements may exist in the financial statements. C. Specified controls requiring segregation of duties may be circumvented by collusion. D. Entity policies may be overridden by senior management.

B. Material misstatements may exist in the financial statements

The ultimate purpose of understanding the entity and its environment and assessing the risks of material misstatement is to contribute to the auditor's assessment of the risk that A. Tests of controls may fail to identify procedures relevant to assertions. B. Material misstatements may exist in the financial statements. C. Specified controls requiring segregation of duties may be circumvented by collusion. D. Entity policies may be inappropriately overridden by senior management.

B. Material misstatements may exist in the financial statements

The audit risk against which the auditor and those who rely on his or her opinion require reasonable protection is a combination of two separate risks at the assertion level. The first risk (consisting of inherent risk and control risk) is that balances, classes of transactions, or disclosures contain material misstatements. The second is that A. The auditor will reject a correct account balance as incorrect. B. Material misstatements that occur will not be detected by the audit. C. The auditor will apply an inappropriate audit procedure. D. The auditor will apply an inappropriate measure of audit materiality.

B. Material misstatements that occur will not be detected by the audit

As the acceptable level of detection risk decreases, an auditor may change the A. Timing of substantive tests by performing them at an interim date rather than at year-end. B. Nature of substantive procedures from a less effective to a more effective procedure. C. Timing of tests of controls by performing them at several dates rather than at one time. D. Assessed level of inherent risk to a higher amount.

B. Nature of substantive procedures from a less effective to more effective procedure

The most likely reason the audit cannot reasonably be expected to bring all noncompliance with laws and regulations by the client to the auditor's attention is that A. Noncompliance is perpetrated by management override of the information and communication component of internal control. B. Noncompliance by clients often relates to operating aspects rather than accounting aspects. C. The information and communication component of the client's internal control may be so effective that the auditor performs only minimal substantive testing. D. Noncompliance may be attributed to the only person in the client's organization with access to both assets and the accounting records.

B. Noncompliance by clients often relates to operating aspects rather than accounting aspects

An auditor wishes to evaluate the design and perform tests of controls over a client's cash disbursements procedures. If the controls leave no audit trail of documentary evidence, the auditor most likely will test the procedures by A. Confirmation and observation. B. Observation and inquiry. C. Analytical procedures and confirmation. D. Inquiry and analytical procedures.

B. Observation and inquiry

Audit evidence concerning undocumented monitoring controls ordinarily is best obtained by A. Performing tests of transactions that corroborate management's financial statement assertions. B. Observing the employees as they apply controls. C. Obtaining a flowchart of activities performed by available personnel. D. Developing audit objectives that reduce control risk.

B. Observing the employees as they apply controls

Most of the auditor's work in forming an opinion on financial statements consists of A. Understanding internal control. B. Obtaining and evaluating audit evidence. C. Examining cash transactions. D. Comparing recorded accountability with assets.

B. Obtaining and evaluating audit evidence

The auditor should perform tests of controls when the auditor's risk assessment includes an expectation A. Of a low level of inherent risk. B. Of the operating effectiveness of internal control. C. That the controls are not suitably designed. D. That the controls are not being applied

B. Of the operating effectiveness of internal control

Control activities include physical controls over access to and use of assets and records. A departure from the purpose of such procedures is that A. Access to the safe-deposit box requires two officers. B. Only storeroom personnel and line supervisors have access to the raw materials storeroom. C. The mail clerk compiles a list of the checks received in the incoming mail. D. Only salespersons and sales supervisors use sales department vehicles.

B. Only storeroom personnel and line supervisors have access to the raw materials storeroom

Which of the following statements about materiality is most likely to be true? A. Materiality requires that relatively more time be directed to those areas that are more susceptible to fraud. B. Performance materiality is less than materiality for the financial statements as a whole. C. Materiality at the assertion level is larger than for the financial statements as a whole. D. Materiality is measured according to specific AICPA standards

B. Performance materiality is less than materiality for the financial statements as a whole

Which of the following is not a medium that can normally be used by an auditor to record information concerning internal control? A. Narrative memorandum. B. Procedures manual. C. Flowchart. D. Decision table.

B. Procedures manual

An auditor should obtain an understanding of an entity's information system, including A. Safeguards used to limit access to computer facilities. B. Process used to prepare significant accounting estimates. C. Procedures used to ensure the proper supervision of staff. D. Programs and controls intended to address the risks of fraud.

B. Process used to prepare significant accounting estimates

Three conditions are generally present in the client's organization when fraud occurs. Those conditions include each of the following except a(n) A. Incentive or pressure to commit fraud. B. Professional skepticism about the likelihood of fraud. C. Opportunity to commit fraud. D. Attitude or rationalization about the act of fraud.

B. Professional skepticism about the likelihood of fraud

Organizational charts are useful to an independent external auditor because they A. Depict all lines of organizational communication. B. Provide a starting point for assessing the risk of material misstatement. C. Ensure the proper division of responsibilities. D. Are essential to effective internal control.

B. Provide a starting point for assessing the risk of material misstatement

An advantage of using systems flowcharts to document information about internal control instead of using internal control questionnaires is that systems flowcharts A. Identify internal control deficiencies more prominently. B. Provide a visual depiction of clients' activities. C. Indicate whether controls are operating effectively. D. Reduce the need to observe clients' employees performing routine tasks.

B. Provide a visual depiction of clients' activities

The primary reason to establish internal control is to A. Safeguard the resources of the organization. B. Provide reasonable assurance that the objectives of the organization are achieved. C. Encourage compliance with organizational objectives. D. Ensure the accuracy, reliability, and timeliness of information.

B. Provide reasonable assurance that the objectives of the organization are achieved

To obtain an understanding of a continuing client in planning an audit, an auditor most likely would A. Perform tests of details of transactions and balances. B. Read internal audit reports. C. Read specialized industry journals. D. Reevaluate the risks of material misstatement.

B. Read internal audit reports

In gathering evidence in the performance of substantive procedures, the auditor most likely A. Uses the test month approach. B. Relies on persuasive rather than conclusive evidence in the majority of cases. C. Considers the client's documentary evidence less reliable than evidence gathered orally by inquiry of management. D. Expresses an adverse opinion if (s)he has substantial doubt as to any assertion of material significance.

B. Relies on persuasive rather than conclusive evidence in the majority of cases

Moor, CPA, discovers a likely fraud during an audit but concludes that its effects, if any, could not be so material as to affect the opinion. Moor most likely should A. Perform additional audit procedures to establish that fraud has occurred. B. Report the finding to the appropriate representatives of the client with the recommendation that it be pursued to a conclusion. C. Confer with the client about the additional audit procedures necessary to establish that fraud has occurred. D. Notify the proper external authorities.

B. Report the finding to the appropriate representatives of the client with the recommendation that it be pursued to a conclusion

Internal control is a function of management, and effective control is based upon the concept of charge and discharge of responsibility and duty. Which of the following is one of the overriding principles of internal control? A. Responsibility for accounting and financial duties should be assigned to one responsible officer. B. Responsibility for the performance of each duty must be fixed. C. Responsibility for the accounting duties must be borne by the audit committee of the company. D. Responsibility for accounting activities and duties must be assigned only to employees who are bonded.

B. Responsibility for the performance of each duty must be fixed

The audit work performed by each assistant should be reviewed to determine whether it was adequately performed and to evaluate whether the A. Auditor's system of quality control has been maintained at a high level. B. Results are consistent with the conclusions to be presented in the auditor's report. C. Audit procedures performed are approved in the professional standards. D. Audit has been performed by persons having appropriate competence and capabilities.

B. Results are consistent with the conclusions to be presented in the auditor's report

Ajax, Inc., is an affiliate of the audit client and is audited by another firm of auditors. Which of the following is most likely to be used by the auditor of the client to obtain assurance that all guarantees of the affiliate's indebtedness have been detected? A. Send the standard bank confirmation request to all of the client's lender banks. B. Review client minutes and obtain a representation letter. C. Examine supporting documents for all entries in intercompany accounts. D. Obtain written confirmation of indebtedness from the auditor of the affiliate.

B. Review client minutes and obtain a representation letter

Assuming a low assessed risk of material misstatement, which of the following audit procedures would be least likely to be performed? A. Physical inspection of a sample of inventory. B. Search for unrecorded cash receipts. C. Obtaining a client representation letter. D. Confirmation of accounts receivable.

B. Search for unrecorded cash receipts

Which of the following statements is most accurate regarding sufficient and appropriate documentation? A. Accounting estimates are not considered sufficient and appropriate documentation. B. Sufficient and appropriate documentation should include evidence that it has been reviewed. C. If additional evidence is required to document significant findings or issues, the original evidence is not considered sufficient and appropriate and therefore should be deleted from the working papers. D. Audit documentation is the property of the client, and sufficient and appropriate copies should be retained by the auditor for at least 5 years.

B. Sufficient and appropriate documentation should include evidence that it has been reviewed

The graphic portrayal of the flow of data and the information processing of a system, including computer hardware, is best displayed in a A. Data-flow diagram. B. System flowchart. C. Gantt chart. D. Program flowchart.

B. System flowchart

An audit in accordance with GAAS is most likely to include comprehensive audit procedures designed to detect material noncompliance by the client relating to A. Environmental laws. B. Tax laws. C. Antitrust laws. D. Insider trading rules.

B. Tax laws

Which of the following ultimately determines the specific audit procedures necessary to provide an independent auditor with a reasonable basis for the expression of an opinion? A. The audit plan. B. The auditor's judgment. C. Auditing standards. D. The audit documentation.

B. The auditor's judgement

Which of the following would an auditor most likely use in determining the auditor's preliminary judgment about materiality for the financial statements as a whole? A. The anticipated sample size of the planned substantive procedures. B. The entity's year-to-date financial results and position. C. The results of the internal control questionnaire. D. The contents of the representation letter.

B. The entity's year-to-date financial results and position

Although substantive tests may support the accuracy of underlying information used in monitoring, these tests may provide no affirmative evidence of the effectiveness of monitoring controls because A. Substantive tests rarely guarantee the accuracy of information used in monitoring if only a sample has been tested. B. The information used in monitoring may be accurate even though it is subject to ineffective control. C. Substantive tests relate to the entire period under audit, but tests of controls ordinarily are confined to the period during which the auditor is on the client's premises. D. When procedures are computerized and leave no audit trail to indicate who performed them, substantive tests may necessarily be limited to inquiries and observation.

B. The information used in monitoring may be accurate even though it is subject to ineffective control

The frequency of the comparison of recorded accountability with assets (for the purpose of safeguarding assets) should be determined by A. The amount of assets independent of the cost of the comparison. B. The nature and amount of the asset and the cost of making the comparison. C. The cost of the comparison and whether the susceptibility to loss results from errors or fraud. D. The auditor in consultation with client management.

B. The nature and amount of the asset and the cost of making the comparison

Auditors may use positive or negative forms of confirmation requests. An auditor most likely will use A. The positive form to confirm all balances regardless of size. B. The negative form for small balances. C. A combination of the two forms, with the positive form used for trade balances and the negative form for other balances. D. The positive form when the assessed risk of material misstatement is acceptably low and the negative form when it is unacceptably high.

B. The negative form for small balances

The concepts of audit risk and materiality are interrelated and must be considered together by the auditor. Which of the following is true? A. Audit risk is the risk that the auditor may unknowingly express a modified opinion when, in fact, the financial statements are fairly stated. B. The phrase in the auditor's report "present fairly, in all material respects, in accordance with accounting principles generally accepted in the United States of America" indicates the auditor's belief that the financial statements as a whole are not materially misstated C. If misstatements are not important individually but are important in the aggregate, the concept of materiality does not apply. D. Material fraud but not material errors cause financial statements to be materially misstated.

B. The phrase in the auditor's report "present fairly, in all material respects, in accordance with accounting principles generally accepted in the United States of America" indicates the auditor's belief that the financial statements as a whole are not materially misstated

In performing tests of controls, the auditor will normally find that A. The level of inherent risk is directly proportional to the rate of error. B. The rate of deviations in the sample exceeds the rate of error in the accounting records. C. The rate of error in the sample exceeds the rate of deviations. D. All unexamined items result in errors in the accounting records.

B. The rate of deviations in the sample exceeds the rate of error in the accounting records

Which of the following statements about audit evidence is true? A. To be appropriate, audit evidence should be either persuasive or relevant but need not be both. B. The sufficiency and appropriateness of audit evidence is a matter of professional judgment. C. The difficulty and expense of obtaining audit evidence about an account balance is a valid basis for omitting the test. D. A client's accounting records can be sufficient audit evidence to support the financial statements.

B. The sufficiency and appropriateness of audit evidence is a matter of professional judgment

Which of the following conditions most likely would pose the greatest risk in accepting a new audit engagement? A. Staff will need to be rescheduled to cover this new client B. There will be a client-imposed scope limitation C. The firm will have to hire a specialist in one audit area D. The client's financial reporting system has been in place for 10 years

B. There will be a client-imposed scope limitation

An auditor might consider the procedures performed by the internal auditors because A. They are employees whose work must be reviewed during substantive testing. B. They are employees whose work may affect the nature, timing, and extent of audit procedures. C. Their work affects the cost-benefit trade-off. D. Their degree of independence may be inferred from the nature of their work.

B. They are employees whose work may affect the nature, timing, and extent of the audit procedures

When expressing an unmodified opinion, the auditor who evaluates the audit findings should determine whether A. The amount of identified misstatement is documented in the management representation letter. B. Uncorrected misstatements are material. C. The amount of identified misstatement is acknowledged and recorded by the client. D. Estimates of total misstatement include the amounts of adjusting entries already recorded by the client.

B. Uncorrected misstatements are material

Auditors sometimes use comparison of ratios as audit evidence. For example, an unexplained decrease in the ratio of gross profit to sales suggests which of the following possibilities? A. Unrecorded purchases. B. Unrecorded sales. C. Merchandise purchases being charged to selling and general expense. D. Fictitious sales.

B. Unrecorded sales

Which of the following circumstances is most likely to cause an auditor to change an assessment of the risk of material misstatement of the financial statements due to fraud? A. Property and equipment are usually sold at a loss before being fully depreciated. B. Unusual discrepancies between the entity's records and confirmation replies. C. Monthly bank reconciliations usually include several in-transit items. D. Clerical errors are listed on a computer-generated exception report.

B. Unusual discrepancies between the entity's records and confirmation replies

The symbol used to represent the file of hard-copy, computer-generated payroll reports kept for future reference is A. Circle with an Extra Line at the Bottom B. Upside Down Triangle with a Line by the Point C. Rectangle with a Pointed Left Side & Indented Right Side D. Slanted Rectangle

B. Upside Down Triangle with a Line by the Point

During the audit of fair value estimates and disclosures, the auditor most likely should A. Understand the components of internal control but need not specifically obtain an understanding of the entity's process for determining fair value estimates. B. Use the understanding of the audited entity's process for determining fair value estimates to assess the risks of material misstatement. C. Determine that the entity has measured fair value estimates using discounted cash flows whenever feasible. D. Focus primarily on the initial recording of transactions.

B. Use the understanding of the audited entity's process for determining fair value estimates to assess the risks of material misstatement

Which of the following is the most reliable analytical approach to verification of the year-end financial statement balances of a wholesale business? A. Verify depreciation expense by multiplying the depreciable asset balances by one divided by the depreciation rate. B. Verify commission expense by multiplying sales revenue by the company's standard commission rate. C. Verify interest expense, which includes imputed interest, by multiplying noncurrent debt balances by the year-end prevailing interest rate. D. Verify FICA tax liability by multiplying total payroll costs by the FICA contribution rate in effect during the year.

B. Verify commission expense by multiplying sales revenue by the company's standard commission rate

To reduce the risks associated with accepting fax responses to requests for confirmations of accounts receivable, an auditor most likely would A. Examine the shipping documents that provide evidence for the existence assertion. B. Verify the sources and contents of the faxes in telephone calls to the senders. C. Consider the faxes to be nonresponses and evaluate them as unadjusted differences. D. Inspect the faxes for forgeries or alterations and consider them to be acceptable if none are noted.

B. Verify the sources and contents of the faxes in telephone calls to the senders

Tests of controls are concerned primarily with each of the following questions except A. How were the controls applied? B. Were the controls approved by the board of directors? C. Were the necessary controls consistently performed? D. By whom were the controls applied?

B. Were the controls approved by the board of directors?

Which of the following statements is an auditor most likely to add to the negative form of confirmation of accounts receivable to encourage timely consideration by the recipient? A. "This is not a request for payment; remittances should not be sent to our auditors in the enclosed envelope." B. "Report any differences on the enclosed statement directly to our auditors; no reply is necessary if this amount agrees with your records." C. "If you do not report any differences within 15 days, it will be assumed that this statement is correct." D. "The following invoices have been selected for confirmation and represent amounts that are overdue."

C. "If you do not report any differences within 15 days, it will be assumed that this statement is correct"

Which of the following documentation is not required for an audit in accordance with auditing standards? A. A written audit plan setting forth the procedures necessary to accomplish the audit's objectives. B. An indication that the accounting records agree or reconcile with the financial statements. C. A client letter that details the auditor's planned field work. D. The basis for the auditor's conclusions about the assessed risks of material misstatement.

C. A client letter that details the auditor's planned field work

The auditor's responsibility for the detection of noncompliance with laws and regulations is greatest for laws and regulations that have A. An indirect effect on financial statement amounts. B. A direct effect on the financial statements that is either material or immaterial. C. A direct effect on the determination of material amounts and disclosures in the financial statements. D. All of the answers are correct.

C. A direct effect on the determination of material amounts and disclosures in the financial statements

Tests of controls in a financial statement audit are least likely to be omitted with regard to A. Accounts believed to be subject to ineffective controls. B. Accounts representing few transactions. C. Accounts representing many transactions. D. Subsequent events.

C. Accounts representing many transactions

Analytical procedures performed to assist in forming an overall conclusion suggest that several accounts have unexpected relationships. The results of these procedures most likely indicate that A. Misstatements exist in the relevant account balances. B. Internal control activities are not operating effectively. C. Additional audit procedures are required. D. The communication with the audit committee should be revised.

C. Additional audit procedures are required

Analytical procedures used as risk assessment procedures should A. Focus on forming an overall conclusion. B. Provide a basis for the opinion. C. Address the risk of material misstatement of revenue due to fraudulent financial reporting. D. Assist in evaluating controls.

C. Address the risk of material misstatement of revenue due to fraudulent financial reporting

Which of the following is an effective audit planning and control procedure that helps prevent misunderstandings and inefficient use of audit personnel? A. Make copies of those client supporting documents examined by the auditor for inclusion in the audit documentation. B. Provide the client with copies of the audit plans to be used during the audit. C. Arrange a preliminary conference with the client to discuss audit objectives, fees, timing, and other information. D. Arrange to have the auditor prepare and post any necessary adjusting or reclassification entries prior to final closing.

C. Arrange a preliminary conference with the client to discuss audit objectives, fees, timing, and other information

When auditing related party transactions, an auditor places primary emphasis on A. Confirming the existence of the related parties. B. Verifying the valuation of the related party transactions. C. Assessing the risks of material misstatement of related party transactions. D. Ascertaining the rights and obligations of the related parties.

C. Assessing the risks of material misstatement of related party transactions

An auditor referred to the findings of an auditor's external specialist in the auditor's report. This may be an appropriate reporting practice if the A. Auditor is not familiar with the professional certification, personal reputation, or particular competence of the specialist. B. Auditor, as a result of the specialist's findings, adds a paragraph emphasizing a matter regarding the financial statements. C. Auditor's report contains a qualified opinion. D. Auditor, as a result of the specialist's findings, decides to indicate a division of responsibility with the specialist for the audit opinion.

C. Auditor's report contains a qualified opinion

When an auditor obtains an understanding of the entity and its environment, including its internal control, which of the following is the most likely order of performing the steps A through C below? A = Tests of controls B = Preparation of a flowchart documenting the understanding of the client's internal control C = Substantive procedures A. ABC. B. ACB. C. BAC. D. BCA.

C. BAC

Which of the following factors does a CPA ordinarily consider in the planning stage of an audit engagement? I. Financial statement accounts likely to contain a misstatement. II. Conditions that require extension of audit tests. A. I only. B. II only. C. Both I and II. D. Neither I nor II.

C. Both I and II

A secondary result of the auditor's understanding of internal control for a nonissuer is that the understanding may A. Provide a basis for determining the nature, timing, and extent of audit tests. B. Assure that management's procedures to detect fraud are properly functioning. C. Bring to the auditor's attention possible control conditions required to be communicated to the client. D. Develop evidence to support the assessed risks of material misstatement.

C. Bring to the auditor's attention possible control conditions required to be communicated to the client

For which of the following transactions would the auditor ordinarily have the greatest difficulty in obtaining assurance that internal control objectives are met? A. Collection of interest and dividends by a retailer. B. Acquisition of production equipment by a manufacturer. C. Collection of contributions from the public by a not-for-profit organization. D. Collection of credit sales by a retailer.

C. Collection of contributions from the public by a not-for-profit organization

The appropriate description that should be placed in symbol D is A. Attach batch total to report and file. B. Reconcile cash balances. C. Compare batch total and correct as necessary. D. Proof report.

C. Compare batch total and correct as necessary

To test the effectiveness of controls, an auditor ordinarily selects from a variety of techniques, including A. Inquiry and analytical procedures. B. Reperformance and observation. C. Comparison and confirmation. D. Inspection and verification.

C. Comparison and confirmation

According to professional standards, analytical procedures are least likely to be applied to A. Test disclosures about reportable operating segments. B. Review financial statements or interim financial information. C. Compile financial statements. D. Assist in forming an overall conclusion.

C. Compile financial statements

Which of the following factors would most likely cause an auditor not to accept a new audit engagement? A. An inadequate understanding of the entity's internal controls. B. The close proximity to the end of the entity's fiscal year. C. Concluding that the entity's management probably lacks integrity. D. An inability to perform preliminary analytical procedures before assessing control risk.

C. Concluding that the entity's management probably lacks integrity

Which of the following procedures is not used in tests of controls over purchases? A. Examine vouchers and supporting documents for authorization. B. Trace vouchers to entries in the vouchers register. C. Confirm inventory held in public warehouses. D. Reperform calculations on some supporting documentation.

C. Confirm inventory held in public warehouses

Before performing substantive analytical procedures at an interim date prior to the balance sheet date, an auditor should A. Obtain audit evidence about the operating effectiveness of controls. B. Determine that the accounts selected for interim testing are not material to the financial statements taken as a whole. C. Consider whether the amounts of the year-end balances selected for interim testing are reasonably predictable. D. Obtain written representations from management that all financial records and related data will be made available.

C. Consider whether the amounts of the year-end balances selected for interim testing are reasonably predictable

During the audit of a new client, the auditor determined that management had given illegal bribes to municipal officials during the year under audit and for several prior years. The auditor notified the client's board of directors, but the board decided to take no action because the amounts involved were immaterial to the financial statements. Under these circumstances, the auditor should A. Add an explanatory paragraph emphasizing that certain matters, while not affecting the unmodified opinion, require disclosure. B. Report the illegal bribes to the municipal official at least one level above those persons who received the bribes. C. Consider withdrawing from the audit engagement and disassociating from future relationships with the client. D. Issue an "except for" qualified opinion or an adverse opinion with a separate paragraph that explains the circumstances.

C. Consider withdrawing from the audit engagement and disassociating from future relationships with the client

The design or operation of a control may not allow management or employees, in the normal course of performing their assigned functions, to prevent, or detect and correct, misstatements on a timely basis. According to AU-C 265, this circumstance is a A. Material weakness. B. Significant deficiency. C. Control deficiency. D. Critical deficiency.

C. Control deficiency

Monitoring of controls is an important component of internal control. Which of the following items is not an example of monitoring? A. Management regularly compares divisional performance with budgets for the division. B. Data processing management regularly generates exception reports for unusual transactions or volumes of transactions and follows up with investigation as to causes. C. Data processing management regularly reconciles batch control totals for items processed with batch controls for items submitted. D. Management has asked internal auditing to perform regular audits of the controls over cash processing.

C. Data processing management regularly reconciles batch control totals for items processed with batch control for items submitted

The diamond-shaped symbol is commonly used in flowcharting to show or represent a A. Process or a single step in a procedure or program. B. Terminal output display. C. Decision point, conditional testing, or branching. D. Predefined process.

C. Decision point, conditional testing, or branching

Audit documentation that records the procedures used by the auditor to gather evidence should be A. Considered the primary support for the financial statements being audited. B. Viewed as the connecting link between the books of account and the financial statements. C. Designed to meet the circumstances of the particular engagement. D. Destroyed when the audited entity ceases to be a client.

C. Designed to meet the circumstances of the particular engagement

The scope and nature of an auditor's contractual obligation to a client is ordinarily set forth in the A. Management representation letter B. Scope paragraph of the auditor's report C. Engagement letter D. Introductory paragraph of the auditor's report

C. Engagement letter

The objective of analytical procedures performed as risk assessment procedures is to A. Evaluate the adequacy of evidence gathered in response to unusual balances identified during the audit. B. Test individual account balances that depend on accounting estimates. C. Enhance the auditor's understanding of the client's business. D. Identify material weaknesses in internal control.

C. Enhance the auditor's understanding of the client's business

After identifying a significant related party transaction outside the entity's normal course of business, an auditor should A. Add an emphasis-of-matter paragraph to the auditor's report to explain the transaction. B. Perform analytical procedures to identify similar transactions that were not recorded. C. Evaluate the business purpose of the transaction. D. Substantiate that the transaction was consummated on terms equivalent to those of an arm's-length transaction.

C. Evaluate the business purpose of the transaction

When an auditor becomes aware of a possible act of noncompliance with laws or regulations, the auditor should obtain an understanding of the nature of the act to A. Consider whether other similar acts have occurred. B. Recommend remedial actions to those charged with governance. C. Evaluate the effect on the financial statements. D. Determine the reliability of management's representations.

C. Evaluate the effect on the financial statements

As part of the audit of fair value estimates and disclosures, an auditor may need to test the entity's significant assumptions. In these circumstances, the auditor should A. Verify that the entity has used its own assumptions, not those of marketplace participants. B. Obtain sufficient evidence to express an opinion on the assumptions. C. Evaluate whether the assumptions individually and as a whole form a reasonable basis for the fair value estimates. D. Apply audit effort equally to all assumptions.

C. Evaluate whether the assumptions individually and as a whole form a reasonable basis for the fair value estimates

The risks of material misstatement (RMMs) should be assessed in terms of A. Specific controls. B. Types of potential fraud. C. Financial statement assertions. D. Control environment factors.

C. Financial statement assertions

The customer checks accompanied by the control tape (refer to symbol A) should be A. Forwarded daily to the billing department for deposit. B. Taken by the mail clerk to the bank for deposit daily. C. Forwarded to the CFO for deposit daily. D. Accumulated for a week and then forwarded to the CFO for deposit weekly.

C. Forwarded to the CFO for deposit daily

A well-designed internal control questionnaire should A. Elicit "yes" or "no" responses rather than narrative responses and be organized by department. B. Be a sufficient source of data for assessment of control risk. C. Help evaluate the effectiveness of internal control. D. Be independent of the objectives of the audit.

C. Help evaluate the effectiveness of internal control

The following are steps in the financial statement audit process: I. Prepare flowchart II. Gather exhibits of all documents III. Interview personnel The most logical sequence of steps is: A. I, II, III. B. I, III, II. C. III, II, I. D. II, I, III.

C. III, II, I

Which of the following is a step in an auditor's decision to rely on internal controls? A. Apply analytical procedures to both financial data and nonfinancial information to detect conditions that may indicate weak controls. B. Perform tests of details of transactions and account balances to identify potential fraud and error. C. Identify specific controls that are likely to prevent, or detect and correct, material misstatements and perform tests of controls. D. Document that the additional audit effort to perform tests of controls exceeds the potential reduction in substantive testing.

C. Identify specific controls that are likely to prevent, or detect and correct, material misstatements and perform tests of controls

In the course of the audit of financial statements for the purpose of expressing an opinion, the auditor will normally prepare a schedule of uncorrected misstatements. The primary purpose served by this schedule is to A. Point out to the responsible entity officials the errors made by various entity personnel. B. Summarize the corrections that must be made before the entity can prepare and submit its federal tax return. C. Identify the potential financial statement effects of misstatements that were not considered clearly trivial when discovered. D. Summarize the misstatements made by the entity so that corrections can be made after the audited financial statements are released.

C. Identify the potential financial statements effects of misstatements that were not considered clearly trivial when discovered

An entity should consider the cost of a control in relationship to the risk. Which of the following controls best reflects this philosophy for a large dollar investment in heavy machine tools? A. Conducting a weekly physical inventory. B. Placing security guards at every entrance 24 hours a day. C. Imprinting a controlled identification number on each tool. D. Having all dispositions approved by the vice president of sales.

C. Imprinting a controlled identification number on each tool

Which of the following is not considered an auditor's specialist? A. Actuary. B. Appraiser. C. Internal auditor. D. Tax attorney.

C. Internal auditor

Observation is considered a reliable audit procedure but one that is limited in usefulness. However, it is used in a number of different audit situations. Which of the following statements is true regarding observation as an audit technique? A. It is the most effective audit methodology to use in filling out internal control questionnaires. B. It is the most persuasive methodology to learn how transactions are really processed during the period under audit. C. It is most persuasive about the performance of a process but is limited to the moment in time at which the observation takes place. D. It is the most persuasive audit technique for determining if fraud has occurred.

C. It is most persuasive about the performance of a process but is limited to the moment in time at which the observation takes place

During the annual audit of Ajax Corp., an issuer, Jones, CPA, a continuing auditor, determined that illegal political contributions had been made during each of the past 7 years, including the year under audit. Jones notified the board of directors about the illegal contributions, but they refused to take any action because the amounts involved were immaterial to the financial statements. Jones should reconsider the intended degree of reliance to be placed on the A. Letter of audit inquiry to the client's attorney. B. Prior years' audit plans. C. Management representation letter. D. Preliminary judgment about materiality levels.

C. Management representation letter

Which of the following is considered a fraudulent activity? A. A mistake in gathering or processing accounting data from which financial statements are prepared. B. An incorrect accounting estimate arising from oversight or misinterpretation of facts. C. Misappropriation of assets. D. A mistake in the application of accounting principles relating to amount, classification, manner of presentation, or disclosure.

C. Misappropriation of assests

When performing procedures to identify and assess the risks of material misstatement for accounting estimates, the auditor should A. Review transactions occurring prior to the date of the auditor's report that indicate variations from expectations. B. Compare independent expectations with recorded estimates to assess management's process. C. Obtain an understanding of how management developed its estimates. D. Analyze historical data used in developing assumptions to determine whether the process is consistent.

C. Obtain an understanding of how management developed its estimates

Analytical procedures are most appropriate when testing which of the following types of transactions? A. Payroll and benefit liabilities. B. Acquisitions and disposals of fixed assets. C. Operating expense transactions. D. Noncurrent debt transactions.

C. Operating expense transactions

Which of the following is an engagement attribute for an audit of an entity that processes most of its financial data in electronic form without any paper documentation? A. Discrete phases of planning, interim, and year-end fieldwork. B. Increased effort to search for evidence of management fraud. C. Performance of audit tests on a continuous basis. D. Increased emphasis on the completeness assertion.

C. Performance of audit tests on a continuous basis

Audit planning for an initial audit most likely includes A. Determining the opinion to be expressed. B. Obtaining an engagement letter prepared by the auditee. C. Performing procedures involving opening balances. D. Selecting a sample of invoices for comparison with shipping reports.

C. Performing procedures involving opening balances

Because of the risk of material misstatement due to fraud, an audit of financial statements in accordance with generally accepted auditing standards should be planned and performed with an attitude of A. Objective judgment. B. Integrity. C. Professional skepticism. D. Impartial conservatism.

C. Professional skepticism

An auditor ordinarily uses a working trial balance resembling the financial statements without notes, but containing columns for A. Cash flow increases and decreases. B. Risk assessments and assertions. C. Reclassifications and adjustments. D. Reconciliations and tick marks.

C. Reclassifications and adjustments

The symbol employed to represent the printing of the employees' paychecks by the computer is A. Upside Down Trapezoid B. Square with the Right Side Open C. Rectangle D. Rectangle with a Slanted Top

C. Rectangle

The auditor is evaluating the effectiveness of a sales commission plan adopted 12 months earlier. An audit procedure likely to provide strong evidence of the plan's effectiveness is to A. Calculate the percentage change in monthly sales by product line for the last 3 years. B. Compare monthly selling costs of this year with those of the 2 preceding years. C. Regress monthly indices of external economic conditions against sales for the 2 preceding years and compare predictions with reported sales. D. Compare the ratio of selling costs per dollar of sales each month for the past year with that of other companies in the industry.

C. Regress monthly indices of external economic conditions against sales for the 2 preceding years and compare predictions with reported sales

Two assertions for which confirmation of accounts receivable balances provides primary evidence are A. Completeness and valuation. B. Valuation and rights and obligations. C. Rights and obligations and existence. D. Existence and completeness.

C. Rights and obligations and existence

Which of the following is an auditor least likely to perform in planning a financial statement audit? A. Coordinating the assistance of entity personnel in data preparation. B. Discussing matters that may affect the audit with firm personnel responsible for non-audit services to the entity. C. Selecting a sample of vendors' invoices for comparison with receiving reports. D. Reading the current year's interim financial statements.

C. Selecting a sample of vendors' invoices for comparison with receiving reports

U.S. GAAP ordinarily require material related party transactions to be A. Accounted for differently from transactions between unrelated parties but not separately disclosed. B. Accounted for on the same basis as transactions between unrelated parties and not separately disclosed. C. Separately disclosed but not accounted for differently from transactions between unrelated parties. D. Separately disclosed and accounted for differently from transactions between unrelated parties.

C. Separately disclosed by not accounted for differently from transactions between unrelated parties

Which of the following nonfinancial information would an auditor most likely consider in performing analytical procedures during the planning phase of an audit? A. Turnover of personnel in the accounting department. B. Objectivity of audit committee members. C. Square footage of selling space. D. Management's plans to repurchase stock.

C. Square footage of selling space

Regardless of the assessed risks of material misstatement, an auditor should perform some A. Tests of controls to determine their effectiveness. B. Analytical procedures to verify the design of controls. C. Substantive procedures to restrict detection risk for significant transaction classes. D. Dual-purpose tests to evaluate both the risk of monetary misstatement and preliminary control risk.

C. Substantive procedures to restrict detection risk for significant transaction classes

Upon discovering material misstatements in a client's financial statements that the client would not revise, a predecessor auditor withdrew from the engagement. If asked by the auditor about the termination of the engagement, the predecessor auditor should A. State that (s)he found material misstatements that the client would not revise B. Suggest that the auditor ask the client C. Suggest that the auditor obtain the client's permission to discuss the reasons D. Indicate that a misunderstanding occurred

C. Suggest that the auditor obtain the client's permission to discuss the reasons

A senior auditor conducted a dual-purpose test on a client's invoice to determine whether the invoice was approved and to ascertain the amount and other terms of the invoice. Which of the following lists two tests that the auditor performed? A. Substantive procedures and analytical procedures. B. Substantive analytical procedures and tests of controls. C. Tests of controls and tests of details. D. Tests of details and substantive procedures.

C. Tests of controls and tests of details

In using the work of an auditor's external specialist, an agreement should exist between the auditor and the specialist as to the nature of the specialist's work. This agreement most likely should include A. A statement that the specialist assumes no responsibility to update the specialist's report for future events or circumstances. B. The conditions under which a division of responsibility may be necessary. C. The applicability of the same confidentiality requirements to the auditor and the specialist. D. The auditor's disclaimer as to whether the specialist's findings corroborate the representations in the financial statements.

C. The applicability of the same confidentiality requirements to the auditor and the specialist

Negative confirmation of accounts receivable is less effective than positive confirmation of accounts receivable because A. A majority of recipients usually lack the willingness to respond objectively. B. Some recipients may report incorrect balances that require extensive follow-up. C. The auditor cannot infer that all nonrespondents have verified their account information. D. Negative confirmations do not produce evidence that is statistically quantifiable.

C. The auditor cannot infer that all nonrespondents have verified their account information

When a management's specialist has assumed full responsibility for taking the client's physical inventory, reliance on the specialist's work is acceptable if A. The auditor is satisfied with the competence of the specialist. B. Circumstances made it impracticable or impossible for the auditor to test the work done by the specialist. C. The auditor conducted the same audit tests and procedures as would have been applicable if the client employees took the physical inventory. D. The auditor's report contains a reference to the assumption of full responsibility by the specialist

C. The auditor conducted the same audit tests and procedures as would have been applicable if the client employees took the physical inventory

A financial statement auditor is considering internal control for a client with an information system that makes extensive use of information technology. Which of the following statements related to the understanding of internal control for this client is false? A. A lack of control at a single user entry point might compromise the security of a single database. B. The auditor may find it necessary to have an expectation of the operating effectiveness of controls for certain relevant assertions. C. The auditor must possess all the information technology skills necessary to complete the engagement. D. Because of the inherent consistency of computer processing, the auditor may be able to reduce the extent of testing an automated control.

C. The auditor must possess all the information technology skills necessary to complete the engagement

In a financial statement audit, A. Regular audit procedures can reasonably be expected to detect all acts of noncompliance with laws and regulations. B. The auditor should contact enforcement agencies when an act of noncompliance with laws and regulations is discovered. C. The auditor should inquire of management about violations of laws and regulations as well as inspect correspondence with regulatory authorities. D. Violations of laws having indirect effects on the financial statements are not of interest to the auditor.

C. The auditor should inquire of management about violations of laws and regulations as well as inspect correspondence with regulatory authorities

An auditor is required to establish an understanding with a client regarding the services to be performed for each engagement. For an auditor of a nonissuer, this understanding generally includes A. The auditor's responsibility for determining the preliminary judgments about materiality and audit risk factors. B. Management's responsibility for identifying mitigating factors when the auditor has doubt about the entity's ability to continue as a going concern. C. The auditor's responsibility for ensuring that management and those charged with governance are away of any significant deficiencies or material weaknesses in control that come to the auditor's attention. D. Management's responsibility for providing the auditor with an assessment of the risks of material misstatement due to fraud.

C. The auditor's responsibility for ensuring that management and those charged with governance are away of any significant deficiencies or material weaknesses in control that come to the auditor's attention.

Confirmation of accounts receivable is a generally accepted auditing procedure. The presumption is that an auditor will request confirmation of accounts receivable. Confirmation is necessary when A. Based on prior years' audit experience, response rates will be inadequate. B. Based on experience with similar engagements, responses are expected to be unreliable. C. The combined assessed level of inherent and control risk is high. D. The accounts receivable are immaterial.

C. The combined assessed level of inherent and control risk is high

Internal control can provide only reasonable assurance of achieving an entity's control objectives. The likelihood of achieving those objectives is affected by which limitation inherent to internal control? A. The auditor's primary responsibility is the detection of fraud. B. The board of directors is active and independent. C. The cost of internal control should not exceed its benefits. D. Management monitors internal control.

C. The cost of internal control should not exceed its benefits

AU-C 500 describes five generalizations about the reliability of evidence. The situations given below indicate the relative degrees of assurance provided by two types of evidence obtained in different situations. Which describes an exception to one of the generalizations? A. The auditor has obtained greater assurance about the balance of sales at Plant A, where (s)he has made limited tests of details because of effective internal control, than at Plant B, where (s)he has made extensive tests of details because of ineffective internal control. B. The auditor's computation of interest payable on outstanding bonds provides greater assurance than reliance on the client's calculation. C. The report of an auditor's specialist regarding the valuation of a collection of paintings held as an investment provides greater assurance than the auditor's physical observation of the paintings. D. The schedule of insurance coverage obtained from the company's insurance agent provides greater assurance than one prepared by the internal audit staff.

C. The report of an auditor's specialist regarding the valuation of a collection of paintings held as an investment provides greater assurance than the auditor's physical observation of the paintings

The independent auditor should understand the internal audit function as it relates to internal control because A. The audit programs, audit documentation, and reports of internal auditors may often be used as a substitute for the work of the independent auditor's staff. B. The procedures performed by the internal audit staff may eliminate the independent auditor's need for considering internal control. C. The work performed by internal auditors may be a factor in determining the nature, timing, and extent of the independent auditor's procedures. D. The understanding of the internal audit function is an important substantive test to be performed by the independent auditor.

C. The work performed by internal auditors may be a factor in determining the nature, timing, and extent of the independent auditor's procedures

Which of the following procedures is the auditor most likely to perform after accepting an initial audit engagement? A. Prepare a rough draft of the financial statement and of the auditor's report. B. Assess control risk for the assertions embodied in the financial statements. C. Tour the client's facilities. D. Consult with and review the work of the predecessor auditor prior to discussing the engagement with the client management.

C. Tour the client's facilities

When assessing the risks of material misstatement at a low level, an auditor is required to document the auditor's Understanding of the Entity's Control Environment, Overall Responses to Assessed Risks A. Yes and No B. No and Yes C. Yes and Yes D. No and No

C. Yes and Yes

Which of the following statements about evidence is true? A. Appropriate evidence supporting management's assertions should be conclusive rather than merely persuasive. B. Effective internal control contributes little to the reliability of the evidence created within the entity. C. The cost of obtaining evidence is not an important consideration to an auditor in deciding what evidence should be obtained. D. A client's accounting records cannot be considered sufficient appropriate audit evidence on which to base the auditor's opinion.

D. A client's accounting records cannot be considered sufficient appropriate audit evidence on which to base the auditor's opinion

In confirming accounts receivable, an auditor decided to confirm customers' account balances rather than individual invoices. Which of the following most likely will be included with the client's confirmation letter? A. An auditor-prepared letter explaining that a nonresponse may cause an inference that the account balance is correct. B. A client-prepared letter reminding the customer that a nonresponse will cause a second request to be sent. C. An auditor-prepared letter requesting the customer to supply missing and incorrect information directly to the client. D. A client-prepared statement of account showing the details of the customer's account balance

D. A client-prepared statement of account showing the details of the customer's account balance

Which of the following statements about internal control is true? A. Properly maintained internal control reasonably ensures that collusion among employees cannot occur. B. The establishment and maintenance of internal control are important responsibilities of the internal auditor. C. Exceptionally effective internal control is enough for the auditor to eliminate substantive procedures on a significant account balance. D. A limitation of internal control is that management makes judgments about the extent of controls it implements.

D. A limitation of internal control is that management makes judgments about the extent of controls it implements

Which of the following is usually included or shown in the audit documentation? A. The procedures used by the auditor to verify the personal financial status of members of the client's management team. B. Analyses that are designed to be a part of, or a substitute for, the client's accounting records. C. Excerpts from authoritative pronouncements that support the underlying generally accepted accounting principles used in preparing the financial statements. D. A summary of how significant findings were addressed.

D. A summary of how significant findings were addressed

Which of the following statements would least likely appear in an auditor's engagement letter? A. Fees for our services are based on our regular per diem rates, plus travel and other out-of-pocket expenses. B. Management is responsible for making all financial records and related information available to us. C. Our engagement is subject to the risk that material fraud or errors, if they exist, will not be detected. D. After performing our preliminary analytical procedures, we will discuss with you the other procedures we consider necessary to complete the engagement.

D. After performing our preliminary analytical procedures, we will discuss with you the other procedures we consider necessary to complete the engagement.

Which of the following statements reflects an auditor's responsibility for detecting fraud and errors? A. An auditor is responsible for detecting employee errors and simple fraud, but not for discovering fraudulent acts involving employee collusion or management override. B. An auditor should plan the audit to detect errors and fraud that are caused by departures from the applicable financial reporting framework. C. An auditor is not responsible for detecting fraud unless the application of GAAS would result in such detection. D. An auditor should design the audit to provide reasonable assurance of detecting fraud and errors that are material to the financial statements.

D. An auditor should design the audit to provide reasonable assurance of detecting fraud and errors that are material to the financial statements

Control activities constitute one of the five components of internal control described in the COSO model. Control activities do not encompass A. Performance reviews. B. Information processing. C. Physical controls. D. An internal auditing function.

D. An internal auditing function

Although the quantity and content of audit documentation vary with each engagement, an auditor's permanent files most likely include A. Schedules that support the current year's adjusting entries. B. Prior years' accounts receivable confirmations that were classified as exceptions. C. Documentation indicating that the audit work was adequately planned and supervised. D. Analyses of capital stock and other owners' equity accounts.

D. Analyses of capital stock and other owner's equity accounts

Which of the following in a predecessor's audit documentation is the auditor least likely to be interested in reviewing? A. Analysis of noncurrent balance sheet accounts. B. Analysis of current balance sheet accounts. C. Analysis of contingencies. D. Analysis of income statement accounts.

D. Analysis of income statement accounts

Which of the following statements about analytical procedures is true? A. Analytical procedures may be omitted entirely for some financial statement audits. B. Analytical procedures used as risk assessment procedures should not use nonfinancial information. C. Analytical procedures usually are effective and efficient for tests of controls. D. Analytical procedures alone may provide the appropriate level of assurance for some assertions.

D. Analytical procedures alone may provide the appropriate level of assurance for some assertions

The most reliable forms of documentary evidence are those documents that are A. Prenumbered. B. Internally generated. C. Easily duplicated. D. Authorized by a responsible official.

D. Authorized by a responsible official

An auditor has withdrawn from an audit engagement of an issuer after finding fraud that may materially affect the financial statements. The auditor should set forth the reasons and findings in communication to the A. PCAOB. B. Client's legal counsel. C. Stock exchanges where the company's stock is traded. D. Board of directors.

D. Board of directors

The symbol used to represent the employees' payroll records stored on magnetic tape is A. Upside Down Triangle with a Line by the Point B. Rectangle with a Pointed Left Side & Indented Right Side C. Squiggly Rectangle D. Circle with an Extra Line at the Bottom

D. Circle with an Extra Line at the Bottom

Which of the following procedures will an auditor most likely perform when evaluating audit evidence at the completion of the audit? A. Obtain assurance from the entity's attorney that all material litigation has been disclosed in the financial statements. B. Verify the clerical accuracy of the entity's proof of cash and its bank cutoff statement. C. Determine whether inadequate provisions for the safeguarding of assets have been corrected. D. Consider whether the results of audit procedures affect the assessment of the identified risks of material misstatement due to fraud.

D. Consider whether the results of audit procedures affect the assessment of the identified risks of material misstatement due to fraud

After obtaining an understanding of the entity and its environment, including its internal control, the auditor assesses A. The need to apply auditing standards. B. Detection risk to determine the acceptable level of inherent risk. C. Detection risk and inherent risk to determine the acceptable level of control risk. D. Control risk and inherent risk to determine the acceptable level of detection risk.

D. Control risk and inherent risk to determine the acceptable level of detection risk

In connection with the consideration of internal control, an auditor encounters the following flow-charting symbols: Upside Down Trapezoid -> Rectangle with a Squiggly Bottom Line The auditor should conclude that a A. Master file has been created by a manual operation. B. Master file has been created by a computer operation. C. Document has been generated by a computer operation. D. Document has been generated by a manual operation.

D. Document has been generated by a manual operation

The objective of tests of details of transactions performed as tests of controls is to A. Monitor the design and use of entity documents such as prenumbered shipping forms. B. Determine whether internal controls have been implemented. C. Detect material misstatements in the account balances of the financial statements. D. Evaluate whether internal controls operated effectively.

D. Evaluate whether internal controls operated effectively

A difference of opinion concerning accounting and auditing matters relative to a particular phase of the audit arises between an assistant auditor and the auditor responsible for the engagement. After appropriate consultation, the assistant auditor asks to be dissociated from the resolution of the matter. The audit documentation would probably be A. Silent on the matter because it is an internal matter of the auditing firm. B. Expanded to note that the assistant auditor is completely dissociated from responsibility for the auditor's opinion. C. Expanded to record the additional work required because all disagreements of this type will require expanded substantive testing. D. Expanded to detail the assistant auditor's position and how the difference of opinion was resolved.

D. Expanded to detail the assistant auditor's position and how the difference of opinion was resolved

In developing written audit plans, an auditor should design specific audit procedures that relate primarily to the A. Timing of the audit. B. Costs and benefits of gathering evidence. C. Financial statements as a whole. D. Financial statement assertions.

D. Financial statement assertions

A document flowchart represents the A. Sequence of logical operations performed during the execution of a computer program. B. Possible combinations of alternative logic conditions and corresponding courses of action for each condition in a computer program. C. Flow of data through a series of operations in an automated data processing system. D. Flow of forms that relate to a particular transaction through an organization.

D. Flow of forms that relate to a particular transaction through an organization

The organization chart is a graphic representation of the A. Power structure. B. Communications channels. C. Locus of decision making. D. Formal authority structure.

D. Formal authority structure

Disclosure of possible fraud to parties other than the client's senior management and those charged with governance ordinarily is not part of an auditor's responsibility. However, to which of the following outside parties may a duty to disclose possible fraud exist? I. To the SEC when the client reports an auditor change II. To a successor auditor when the successor makes appropriate inquiries III. To a government funding agency from which the client receives financial assistance A. I and II. B. I and III. C. II and III. D. I, II, and III.

D. I, II, and III

An auditor most likely obtains an understanding of a new client to A. Make constructive suggestions concerning improvements to the client's internal control. B. Develop an attitude of professional skepticism concerning management's financial statement assertions. C. Evaluate whether the aggregation of known misstatements causes the financial statements taken as a whole to be materially misstated. D. Identify areas of audit emphasis.

D. Identify areas of audit emphasis

Some account balances, such as those for pensions or leases, are the results of complex calculations. The susceptibility to material misstatements in these types of accounts is defined as A. Audit risk. B. Detection risk. C. Sampling risk. D. Inherent risk.

D. Inherent risk

Which of the following procedures would provide the most reliable audit evidence? A. Inquiries of the client's internal audit staff held in private. B. Inspection of prenumbered client purchase orders filed in the vouchers payable department. C. Analytical procedures performed by the auditor on the entity's trial balance. D. Inspection of bank statements obtained directly from the client's financial institution.

D. Inspection of bank statements obtained directly from the client's financial institution

During the consideration of fraud in a financial statement audit, the auditor should identify and assess risks that may result in material misstatements due to fraud. This assessment A. Must state an overall judgment about whether an identified risk is high, medium, or low. B. Requires an observation that the three fraud conditions are present. C. Follows the auditor's determination that the related controls are operating effectively. D. Is based on evaluating whether the entity's related controls have been suitably designed and implemented.

D. Is based on evaluating whether the entity's related controls have been suitably designed and implemented

With respect to the auditor's planning of a year-end audit, which of the following statements is always true? A. An engagement should not be accepted after the fiscal year-end. B. An inventory count must be observed at the balance sheet date. C. Those charged with governance should not be told of the specific audit procedures that were performed. D. It is an acceptable practice to carry out parts of the examination at interim dates.

D. It is an acceptable practice to carry out parts of the examination at interim dates.

The audit working paper that reflects the major components of an amount reported in the financial statements is the A. Interbank transfer schedule. B. Carryforward schedule. C. Supporting schedule. D. Lead schedule.

D. Lead schedule

Which of the following circumstances would an auditor most likely consider a risk factor relating to misstatements arising from fraudulent financial reporting? A. Several members of management have recently purchased additional shares of the entity's stock. B. Several members of the board of directors have recently sold shares of the entity's stock. C. The entity distributes financial forecasts to financial analysts that predict conservative operating results. D. Management is interested in maintaining the entity's earnings trend by using aggressive accounting practices.

D. Management is interested in maintaining the entity's earnings trend by using aggressive accounting practices

The correct labeling, in order, for the following flowchart symbols is Upside Down Trapezoid, Rectangle with a Squiggly Bottom Line, Rectangle with a Pointed Left Side & Indented Right Side, Rectangle A. Document, display, online storage, and entry operation. B. Manual operation, processing, offline storage, and input-output activity. C. Display, document, online storage, and entry operation. D. Manual operation, document, online storage, and entry operation.

D. Manual operation, document, online storage, and entry operation

In the absence of evidence to the contrary, transactions with related parties should not be assumed to be outside the normal course of business. The auditor should, however, be aware of the possibility that transactions with related parties may have been motivated solely or in large part by extraordinary conditions. Which of the following is not normally a condition motivating a transaction outside the normal course of business? A. Lack of sufficient working capital or credit to continue business. B. An overly optimistic earnings forecast. C. Dependence on a single or relatively few product(s), customer(s), or transaction(s) for the ongoing success of the venture. D. Mutual benefit to both parties.

D. Mutual benefit to both parties

For which of the following judgments may an independent auditor share responsibility with an entity's internal auditor who is assessed to be both competent and objective? Materiality of Evaluation of Significant, Misstatements Accounting Estimates A. Yes and No B. No and Yes C. Yes and Yes D. No and No

D. No and No

Which of the following statements regarding auditor documentation of the understanding of the client's internal control components obtained to plan the audit is correct? A. Documentation must include flowcharts. B. Documentation must include procedural write-ups. C. No documentation is necessary although it is desirable. D. No one particular form of documentation is necessary, and the extent of documentation may vary.

D. No one particular form of documentation is necessary, and the extent of documentation may vary

As part of understanding internal control relevant to the audit of a non issuer, an auditor does not need to A. Consider factors that affect the risks of material misstatement. B. Determine whether controls have been implemented. C. Identify the risks of material misstatement. D. Obtain knowledge about the operating effectiveness of internal control.

D. Obtain knowledge about the operating effectiveness of internal control

In developing an audit plan, an auditor should A. Determine whether the allowance for sampling risk exceeds the achieved upper precision limit B. Evaluate findings from substantive procedures performed at interim dates C. Consider whether the inquiry of the client's attorney identifies any litigation, claims, or assessments not disclosed in the financial statements D. Perform risk assessment procedures

D. Perform risk assessment procedures

Prior to beginning the field work on a new audit engagement in which a CPA does not possess expertise in the industry in which the client operates, the CPA should A. Reduce audit risk by lowering initial levels of materiality. B. Design special substantive procedures to compensate for the lack of industry expertise. C. Engage financial experts familiar with the nature of the industry. D. Perform risk assessment procedures.

D. Perform risk assessment procedures

Proper segregation of duties reduces the opportunities to allow persons to be in positions both to A. Journalize entries and prepare financial statements. B. Record cash receipts and cash disbursements. C. Establish internal control and authorize transactions. D. Perpetrate and conceal fraud and error.

D. Perpetrate and conceal fraud and error

When documenting internal control, the independent auditor sometimes uses a systems flowchart, which can best be described as a A. Pictorial presentation of the flow of instructions in a client's internal computer system. B. Diagram that clearly indicates an organization's internal reporting structure. C. Graphic illustration of the flow of operations that is used to replace the auditor's internal control questionnaire. D. Symbolic representation of a system or series of sequential processes.

D. Symbolic representation of a system or series of sequential processes

A basic premise underlying analytical procedures is that A. These procedures cannot replace tests of balances and transactions. B. Statistical tests of financial information may lead to the discovery of material misstatements in the financial statements. C. The study of financial ratios is an acceptable alternative to the investigation of unusual fluctuations. D. Plausible relationships among data may reasonably be expected to exist and continue in the absence of known conditions to the contrary.

D. Plausible relationships among data may reasonably be expected to exist and continue in the absence of known conditions to the contrary

Which of the following must an auditor document with respect to the consideration of fraud in a financial statement audit? A. Reasons for not identifying management override as a fraud risk. B. Reasons for not identifying collusion as a fraud risk. C. Instances of the auditor's exercise of professional skepticism during the consideration of fraud. D. Reasons for not identifying improper revenue recognition as a fraud risk.

D. Reasons for not identifying improper revenue recognition as a fraud risk

A proper segregation of duties requires that an individual A. Authorizing a transaction records it. B. Authorizing a transaction maintain custody of the asset that resulted from the transaction. C. Maintaining custody of an asset be entitled to access the accounting records for the asset. D. Recording a transaction not compare the accounting record of the asset with the asset itself.

D. Recording a transaction not compare the accounting record of the asset with the asset itself

The symbol employed to represent the employees' checks printed by the computer is A. Slanted Rectangle B. Upside Down Trapezoid C. Rectangle with a Slanted Top D. Rectangle with a Squiggly Bottom

D. Rectangle with a Squiggly Bottom

Which of the following symbolic representations indicates that a sales invoice has been filed? A. Rectangle with a Squiggly Bottom Line -> Upside Down Trapezoid B. Upside Down Triangle with a Line by the Point -> Upside Down Trapezoid C. Upside Down Trapezoid -> Upside Down Triangle with a Line by the Point D. Rectangle with a Squiggly Bottom Line -> Upside Down Triangle with a Line by the Point

D. Rectangle with a Squiggly Bottom Line -> Upside Down Triangle with a Line by the Point

The symbol used to represent the weekly payroll register generated by the computer is A. Circle with an Extra Line at the Bottom B. Upside Down Triangle with a Line by the Point C. Rectangle D. Rectangle with a Squiggly Line at the Bottom

D. Rectangle with a Squiggly Line at the Bottom

Which of the following circumstances would permit an independent auditor to accept an engagement after the close of the fiscal year? A. Issuance of a disclaimer of opinion as a result of inability to conduct certain tests required by generally accepting auditing standards due to the timing of the acceptance of the engagement. B. An expectation of the effectiveness of internal control C. Receipt of an assertion from the preceding auditor that the entity will be able to continue as a going concern. D. Remedy of limitations resulting from accepting the engagement after the close of the end of the year, such as those relating to the existence of physical inventory.

D. Remedy of limitations resulting from accepting the engagement after the close of the end of the year, such as those relating to the existence of physical inventory

An auditor is planning an audit engagement for a new client in a business that is unfamiliar to the auditor. Which of the following would be the least useful source of information for the auditor during the preliminary planning stage, when the auditor is trying to obtain a general understanding of audit problems that might be encountered? A. Textbooks and periodicals related to the industry. B. AICPA Audit and Accounting Guides. C. Financial statements of other entities in the industry. D. Results of performing substantive procedures.

D. Results of performing substantive procedures

Which of the following procedures will most likely assist an auditor in determining whether management has identified all accounting estimates that could be material to the financial statements? A. Inquire about the existence of related party transactions. B. Determine whether the outcomes of accounting estimates differ from the amounts originally recognized. C. Confirm inventories at locations outside the entity. D. Review the lawyer's letter for information about litigation.

D. Review the lawyer's letter for information about litigation

In order to obtain an initial understanding of internal control sufficient to assess the risk of material misstatement of the financial statements, an auditor would most likely perform which of the following procedures? A. Tests of key controls to determine whether they are effective. B. Expanded substantive testing to identify relevant controls. C. Analytical procedures to determine the need for specific controls. D. Risk-assessment procedures to evaluate the design of relevant controls.

D. Risk-assessment procedures to evaluate the design of relevant controls.

The appropriateness of evidence available to an auditor is least likely to be affected by the A. Relevance of such evidence to the financial statement assertion being investigated. B. Relationship of the preparer of such evidence to the entity being audited. C. Timeliness of such audit evidence. D. Sampling method employed by the auditor to obtain a sample of such evidence.

D. Sampling method employed by the auditor to obtain a sample of such evidence

In obtaining an understanding of internal control in a financial statement audit, an auditor is not obligated to A. Determine whether the controls have been implemented. B. Perform procedures to understand the design of internal control. C. Document the understanding of the entity's internal control components. D. Search for significant deficiencies in the operation of internal control.

D. Search for significant deficiencies in the operation of internal control

In obtaining an understanding of an issuer's internal control, an auditor does all the following except A. Inspect documents. B. Observe employees. C. Perform a walkthrough of the transaction process. D. Send confirmations to customers.

D. Send confirmations to customers

If internal control is properly designed, the same employee may be permitted to A. Receive and deposit checks and also approve write-offs of customer accounts. B. Approve vouchers for payment and also sign checks. C. Reconcile the bank statements and also receive and deposit cash. D. Sign checks and also cancel supporting documents.

D. Sign checks and also cancel supporting documents

Which of the following is a true statement about an auditor's responsibility regarding consideration of fraud in a financial statement audit? A. The auditor should consider the client's internal control and plan and perform the audit to provide absolute assurance of detecting all material misstatements. B. The auditor should assess the risk that errors may cause the financial statements to contain any misstatements and determine whether the necessary controls are prescribed and are being followed satisfactorily. C. The auditor should consider the types of misstatements that could occur and perform tests on 100% of the information subject to misstatement. D. The auditor should assess the risks of material misstatement due to fraud.

D. The auditor should assess the risks of material misstatement due to fraud

Which of the following statements is true about related party transactions? A. In the absence of evidence to the contrary, related party transactions should be assumed to be outside the ordinary course of business. B. An auditor should determine whether a particular transaction would have occurred if the parties had not been related. C. An auditor should substantiate that related party transactions were consummated on terms equivalent to those that prevail in arm's-length transactions. D. The auditor should consider whether an identified related party transaction outside the normal course of business is appropriately accounted for and disclosed.

D. The auditor should consider whether an identified related party transaction outside the normal course of business is appropriately accounted for and disclosed

Which of the following statements is true about the use of the work of an auditor's specialist? A. The specialist need not agree to the auditor's use of the specialist's findings. B. The auditor is required to perform substantive procedures to verify the specialist's assumptions and findings. C. The auditor must keep client information confidential, but the specialist is not obligated to do so. D. The auditor should obtain an understanding of the methods and assumptions used by the specialist.

D. The auditor should obtain an understanding of the methods and assumptions used by the specialist

The auditor should establish an overall audit strategy. Which one of the following statements is most consistent with this requirement? A. The auditor should have appropriate proficiency to perform the audit. B. The auditor must be independent of the client C. The auditor should communicate certain issues to those charged with governance D. The auditor should plan the audit so that it will be performed effectively

D. The auditor should plan the audit so that it will be performed effectively

An auditor searching for related party transactions should obtain an understanding of each subsidiary's relationship to the total entity because A. This may permit the audit of interentity account balances to be performed as of concurrent dates. B. Interentity transactions may have been consummated on terms equivalent to arm's-length transactions. C. This may reveal whether particular transactions would have taken place if the parties had not been related. D. The business structure may be deliberately designed to obscure related party transactions.

D. The business structure may be deliberately designed to obscure related party transactions

Which of the following is a false statement about the relationship of the internal auditor and the scope of the external audit of a company's financial statements? A. The nature, timing, and extent of the external auditor's substantive tests may be affected by the work of the internal auditors. B. The internal auditors may assist the external auditor in performing substantive tests and tests of controls under certain circumstances. C. The external auditor is not required to give consideration to the internal audit function beyond obtaining an understanding sufficient to identify activities relevant to planning the audit. D. The internal auditors may determine the extent to which audit procedures should be employed by the external auditor.

D. The internal auditors may determine the extent to which audit procedures should be employed by the external auditor

In planning the audit engagement, the auditor should consider each of the following except A. The auditor's independence. B. Risks of material misstatement due to fraud. C. Anticipated levels of audit risk and materiality. D. The kind of opinion (unmodified, qualified, or adverse) that is likely to be expressed.

D. The kind of opinion (unmodified, qualified, or adverse) that is likely to be expressed

Based on an understanding of internal control completed at an interim date, the auditor assessed the risks of material misstatement at the relevant assertion level and performed interim substantive procedures. The records will most likely be tested again at year end if A. Tests of controls were not performed by the internal auditor during the remaining period. B. Internal control provides a basis for limiting the extent of substantive testing. C. The auditor used nonstatistical sampling during the interim period testing of controls. D. The remaining period is long.

D. The remaining period is long

In assessing the competence and objectivity of an entity's internal auditor, an independent auditor would least likely consider information obtained from A. Discussions with management personnel. B. External quality reviews of the internal auditor's activities. C. Previous experience with the internal auditor. D. The results of analytical procedures.

D. The results of analytical procedures

Which of the following auditor concerns most likely could be so serious that the auditor concludes that a financial statement audit cannot be performed? A. Management fails to modify prescribed internal controls for changes in information technology. B. Internal control activities requiring segregation of duties are rarely monitored by management. C. Management is dominated by one person who is also the majority shareholder. D. There is a substantial risk of intentional misapplication of accounting principles.

D. There is a substantial risk of intentional misapplication of accounting principles

The element of the audit-planning process most likely to be agreed upon with the client before implementation of the audit strategy is the determination of the A. Evidence to be gathered to provide a sufficient basis for the auditor's opinion. B. Procedures to be undertaken to discover litigation, claims, and assessments. C. Pending legal matters to be included in the inquiry of the client's attorney. D. Timing of inventory observation procedures to be performed.

D. Timing of inventory observation procedures to be performed

Which of the following circumstances most likely will cause an auditor to consider whether material misstatements due to fraud exist in an entity's financial statements? A. Management places little emphasis on meeting earnings projections of external parties. B. The board of directors oversees the financial reporting process and internal control. C. Control deficiencies previously communicated to management are not corrected. D. Transactions selected for testing are not supported by proper documentation.

D. Transactions selected for testing are not supported by proper documentation

During the process of confirming receivables as of December 31, Year 1, a positive confirmation was returned indicating the "balance owed as of December 31 was paid on January 9, Year 2." The auditor would most likely A. Determine whether any changes in the account occurred between January 1 and January 9, Year 2. B. Determine whether a customary trade discount was taken by the customer. C. Reconfirm the zero balance as of January 10, Year 2. D. Verify that the amount was received.

D. Verify that the amount was received

During an audit, an internal auditor may provide direct assistance to an independent CPA in Obtaining an Understanding of Internal Control, Performing Tests of Controls, Performing Substantive Tests A. No, No, and No B. Yes, No, and No C. Yes, Yes, and No D. Yes, Yes, and Yes

D. Yes, Yes, and Yes


Conjuntos de estudio relacionados

Chinese International Relations Vocabulary

View Set

english 12 online answers for final

View Set

Comptia A+ core 2 wrong answers

View Set